AHDI CMT Prep Challenge Quiz/Exam guide

¡Supera tus tareas y exámenes ahora con Quizwiz!

What condition is suggested by a homocysteine level of 50 mcmol/L? A. increased risk of clot formation B. increased risk for ovarian carcinoma C. increased risk of prostatic carcinoma D. increased risk of thyrotoxic crisis

A

What condition is suggested by an abnomal Wood lamp examination? A. fungal infection B. leukemia C. tuberculosis D. anemia

A

What does the "M" in ASTM refer to? A. materials B. management C. medicine D. manual

A

What is SNOMED? A. clinical nomenclature system B. computer-suggested coding program C. integrated data management platform D. performance measurement standard

A

According to the Book of Style, which elements entered or captured in an electronic environment may not translate correctly? A. demographic information B. drug dosages C. report templates D. use of certain symbols and styles

D

An SRT draft document states "no allergies" on page 1 and "allergic to penicillin" on page 3. What step should the editor take? A. delete "no allergies" from page 1 B. leave as originally drafted C. listen to voice file D. flag for physician to verify

D

The "once-and-done" concept is utilized with which of the following? A. point-and-click template systems B. front-end speech recognition C. back-end speech recognition D. at the point of authentication of the record

B

What is the purpose of a VDRL test? A. To evaluate for any STD B. Evaluation for HIV C. To evaluate for syphilis D. Evaluation for gonorrhea

C

Which imaging study uses an iodine-based contrast material? A. virtual colonoscopy B. brain MRI C. intravenous pyelogram D. MRA

C

Sessile means: A attached by a slender stalk B having slender filaments C having a broad base of attachment D flimsy

C Sessile comes from the Latin word sissilis meaning low-growing. A pedunculated polyp is attached by means of a flimsy stalk.

The BNP level, used during evaluation of patients for CHF, measures what? A. a polypeptide secreated by the cardiac ventricles B. average red blood cell production during a specified time frame C. the presence or absence of neurofibrillary tangles D. nitrogen in the blood

A

The Roux-en-Y technique is used in which kind of surgical specialty? A. bariatric B. arthroscopic C. urologic D. ophthalmologic

A

What kind of surgery uses an anterior approach through the sternocleidomastoid muscle? A. inguinal hernia repair B. cervical diskectomy C. SLAP repair D. lumbar fusion

B

What kind of surgery would use a columellar chevron incision? A. angioplasty B. open rhinoplasty C. adenoidectomy D. cataract extraction

B

What procedure would use a #2 cutting block? A. appendectomy B. total knee arthroplasty C. rhinoplasty D. arteriotomy

B

What proportions does an A/G ratio measure? A. albumin and globulin in urine B. albumin and globulin in blood C. hemoglobin A and glucose in blood D. hemoglobin A and glucose in urine

B

What substance is used in performing a barium swallow? A. barium carbonate B. barium sulfate C. barium oxide D. barium peroxide

B

What type of procedure involves a low transverse abdominal incision? A. open appendectomy B. primary cesarean section C. colonoscopy D. Whipple procedure

B

What would be a recommended course of treatment for plantar fasciitis? A. casting and long-term antibiotics B. stretching exercises and anti-inflammatories C. outpatient surgery followed by antibiotics D. outpatient surgery followed by casting

B

Which is the correct expression of a measurement commonly encountered in carotid angiograms? A. 4F B. 4 French C. 4f D. 4 Fr

B

Which ligament runs on the outside of the knee and limits side-to-side motion? A. anterior cruciate B. lateral collateral C. posterior cruciate D. medial collateral

B

Which sentence is correct? A. A drug-eluding stent was used. B. A drug-eluting stent was used. C. A drug-eliding stent was used. D. A drug-alluding stent was used.

B

Which sentence is correctly transcribed? A. Recanalization of both vas deferens had occurred. B. Recanalization of both vasa deferentia had occurred. C. Rechannelization of both vas deferens had occurred. D. Rechannelization of both vas deferentia had occurred.

B

Which term describes the kind of health data created from narrative dictation? A. mineable B. unstructured C. discrete D. abstracted

B

A woman who is para 2-1-0-3 has been pregnant at least: A one time B two times C three times D four times

B A value in the first three indicators of a TPAL designation will indicate the minimum number a times a patient has been pregnant, given the possibility of a twins/multiples pregnancy. When gravida information is unknown, it can be assumed that a para 2-1-0-3 female has been pregnant at least twice (since a twin full-term delivery and a single premature delivery are a reasonable possibility).

The gland that controls functional maturation of lymphocytes is the: A pituitary B thymus C adrenal D thyroid

B Lymphocytes that originate from stem cells in the bone marrow pass through the thymus gland where they mature and develop into activated T-lymphocytes, able to respond to antigens elsewhere in the body.

Which describes the procedure used in a retrograde pyelography? A. contrast is injected into the renal calyces. B. Contrast injected into the urethra is used to visualize the bladder. C. Contrast is injected into the ureters from an endoscope placed in the bladder. D. Contrast injected into the bladder is observed as it passes through the urethra.

C

Which abbreviation refers to an enforced audit being mandated by the Centers for Medicare and Medicaid (CMS) to review improper payments made to healthcare providers? A. MAC B. PQRI C. RAC D. P4P

C

Which condition might be treated with an ileal conduit? A. early-stage colon carcinoma B. avascular necrosis C. absence of the urinary bladder after surgery D. iatrogenic lymphedema

C

Which is a brand of electrocautery device used for hemostasis? A. DePuy B. ElecSys C. Bugbee D. Arctic

C

Which is the correct expression for classifying adenocarcinoma of the prostate? A. Gleason grade 6 B. Gleason stage VI C. Gleason score 6 D. Gleason level 6

C

Which of the following may negatively impact adoption/integration for SRT? A. consistent dictator speech patterns B. high accuracy of draft documents C. hardware/software amortization D. low training costs

C

Which organization, established by the Office of the National Coordinator for Health IT, certifies the products that meet or exceed standards for usability and interoperability? A. HL7 B. HITSP C. CCHIT D. RHIO

C

Which procedure places the patient in a flank position and uses an incision overlying the 12th rib? A. appendectomy B. colectomy C. nephrectomy D. distal pancreatectomy

C

Which refers to the application of computer technology to the management of information? A. integration B. interoperability C. informatics D. implementation

C

Which sentence is correctly transcribed? A. Plain x-ray of the lower extremity showed a fracture of the malleus. B. Plane x-ray of the lower extremity showed a fracture of the malleus. C. Plain x-ray of the lower extremity showed a fracture of the malleolus. D. Plane x-ray of the lower extremity showed a fracture of the malleolus.

C

Which statement about the brain is true? A. The brain consumes about 80% of the oxygen and glucose used in the body. B. Large amounts of glucose are stored in the brain. C. Blood flows to the brain mainly through the internal carotid and vertebral arteries. D. Proteins and antibiotics easily cross the blood-brain barrier.

C

Which abbreviation refers to a common test for allergies? A ANA B AFP C RAST D FTA

C Serologic testing for allergic response is done by radioallergosorbent test (RAST), a type of radioimmunoassay that uses radioisotope tags to allergens in order to detect IgE antibodies to these allergens in the serum of allergic persons.

What medication would be used in the treatment of Lyme disease? A. doxycycline B. aripiprazole C. olmesartan D. omeprazole

A

Which of the following tests is used in combination with a PT (prothrombin time) to assess Coumadin (anticoagulation) therapy? A INR B MCH C d-dimer D fibrinogen

A INR (international normalized ratio) is used to report a standardized result for coagulation studies in patients on Coumadin therapy. Each batch of thromboplastin reagent (the agent used in the testing process) varies slightly, causing the prothrombin time result to vary. To avoid fluctuating PT results, the lab calculates the INR based on a sensitivity index for the particular batch of thromboplastin used. In this way, clinicians are sure to get consistent results for monitoring Coumadin therapy regardless of the lab or equipment used to perform the test. A PT/INR is actually one test (the PT) with results reported in two ways.

What condition might be indicated by the presence of bruises and petechiae on the extremities? A. ITP B. DLE C. ARDS D. MAC

A Idiopathic thrombocytopenic purpura

Seldinger technique refers to which type of procedure? A percutaneous insertion of a catheter B internal fixation of a fractured bone C anastamosis of two vessels D suturing of subcutaneous tissues

A This technique is used to access a blood vessel or space. A needle is used to puncture the structure and a guide wire is threaded through the needle. When the needle is withdrawn, a catheter is threaded over the wire which is also withdrawn, leaving the catheter in place. This technique is seen frequently in cardiac catheterization.

The term marche à petits pas refers to an abnormality of: A gait B speech C sensation D dexterity

A This term is used to describe the frozen or ataxic gait commonly associated with Parkinson disease.

Tumor grade refers specifically to which of these clinico-pathologic findings? A resemblance to the tissue of origin B presence or absence of metastases C degree of local tissue destruction D tumor's response to hormone therapy

A Tumor grading is based on the degree of differentiation, or variability of resemblance to the surrounding tissue, of the tumor itself and is indicative of the progression and aggression of the disease.

During what procedure would a pigtail catheter be used? A. cystectomy B. left heart catheterization C. oophorectomy D. prostatectomy

B

___________ hearing loss refers to hearing that is better during loud noise. A conductive B paradoxical C noise-induced D ototoxic

B Also called paracusis of Willis, paracusia willisiana and paradoxic deafness.

Organisms that cause infections, such as viruses and bacteria, are: A toxins B pathogens C venoms D parasites

B From the root patho, meaning disease

Choose the correctly transcribed sentence. A. Imaging showed an area of plague with vessel remottling. B. Imaging showed an area of plaque with vessel remottling. C. Imaging showed an area of plaque with vessel remodeling. D. Imagining showed an area of plaque with vessel remodeling.

C

In the Binet leukemia classification system, which is the correct expression to describe a case with fewer than 3 areas of enlarge lymphoid tissue? A. Binet grade 1 B. Binet level 1 C. Binet stage A D. Binet stage I

C

In the context of a sleep study, what does the acronym REM stand for? A. regular eye motion B. random eye movement C. rapid eye movement D. repeating eye motion

C

On physical examination, we found asymmetry, irregularity, color variation, and a 3 cm diameter. What is being evaluated? A. a burned area of skin surface B. the iris of the eye C. a lesion suspicious for melanoma D. sclerodermatous skin changes

C

Pick the correct abbreviation that stands for flexion, abduction, external rotation, extension. A. FAERE B. FLABERE C. FABERE D. FAEX

C

The abdominopelvic cavity is divided into how many regions? A 3 B 6 C 9 D 10

C The abdominopelvic cavity is divided into nine regions: right hypochondriac, epigastric, left hypochondriac, right lumbar, umbilical, left lumbar, right iliac, hypogastric, and left iliac.

The inner layer of an arterial wall, or intima, is also known as the: A endothelium B adventitia C tunica interna D internal elastic lamina

C The arterial wall has three coats, or tunics. The inner coat [intima] is also called the tunica interna. The tunica media and tunica externa [adventitia] compose the middle and outer layers.

Which of the following findings would be transcribed correctly on an EKG? A right axes deviation B right excess deviation C right axis deviation D right access deviation

C The axis is the general direction of the electrical impulse through the heart. A normal axis is directed to the bottom left. An extreme right axis deviation may indicate bundle branch block, ventricular hypertrophy, or pulmonary embolism; although the axis may also deviate to the right in very tall or obese patients. There is only one axis, so answer D (axes, the plural of axis) would not be correct.

The basilic veins are located in the: A pelvis B neck C upper extremities D lower extremities

C The brachial and basilic veins unite with the axillary veins of the upper extremities. The axillaries end at the first rib, where they become subclavians.

This 60-year-old woman presents with acute onset of respiratory distress 5 days following colectomy for colon carcinoma. We will order a lung V/Q scan and _____. Which phrase is most likely to fill in the blank? A. begin chest physiotherapy B. initiate antibiotic therapy C. perform immediate bronchoscopy D. consider starting heparin

D

This patient presented with symptoms of intermittent crampy abdominal pain, bilious vomiting, and _____. The differential diagnosis includes mechanical bowel obstruction. Which phrase would be likely to fill in the above blank? A. epigastric tenderness B. marked icterus C. light-colored stool D. a history of laparotomy

D

What is measured when tidal volume is assessed? A. maximum amount of air a patient can exhale after taking a deep breath. B. volume of air that remains in the lungs at all times C. volume of the air that the lungs can hold D. volume of air that can be moved in and out of the lungs during eupnea

D

What is the primary function of NLP technology? A. To capture dictated narrative and create a speech-recognized draft. B. To enable disparate health IT systems to exchange data. C. To format narrative reports in compliance with HL7's CDA standards. D. To codify unstructured narrative against data measurement systems for electronic use and analysis.

D

CT scans use which imaging technology? A Sound waves B Gamma rays C Microwaves D X-rays

D CT (computed tomography) uses x-ray images, taken in thin, progressive "slices" and reconstructed using computer programs to create 3-dimensional images. The word tomography comes from Greek tomos (slice) and graphia (to draw).

A patient's dizziness might be evaluated with what testing? A. indocyanine green angiogram B. radioactive uptake iodine test C. Coombs test D. electronystagmography

D study used to clinically evaluate patients with dizziness, vertigo, or balance dysfunction

A postoperative patient is noted in the recovery room to have indicators of volume depletion including _____ and _____. What are the likely signs or symptoms mentioned in the blanks? A. hypertension and excessive urine output B. hypotension and excessive urine output C. hypotension and low urine output D. hypertension and low urine output

C

A(n) _____ will be ordered as an initial diagnostic step in evaluation of this patient's dominant thyroid nodule. What kind of test or exam is most likely to be performed? A. plain x-ray B. scintigraphy C. fine-needle aspiration biopsy D. PET scan

C

Abadie sign, or spasm of the levator palpebrae superioris, may be a sign of what condition? A. hypothyroidism B. glaucoma C. hyperthyroidism D. senile cataract

C

Coagulation factor X is also known as the ________ factor. A Christmas B Hageman C Stuart D Prothrombin

C There are 12 coagulation factors (they are named I - XIII, but there is no factor VI). Blood factor X is also known as the Stuart factor or Power factor. Factor IX is known as the Christmas factor, and factor XII is known as the Hageman factor or glass factor.

The middle meninx of the spinal meninges is called the: A dura mater B epidural space C arachnoid D pia mater

C There are only three spinal meninges, of which the arachnoid is the middle meninx [singular form of meninges]. It is called this because of its spider's web arrangement of delicate collagen and elastic fibers.

A ________ sign is elicited in acute appendicitis. A Babinski B Brudzinksi C McBurney D Murphy

C This sign, represented by tenderness at the site two-thirds of the distance between the umbilicus and the anterior-superior iliac spine [McBurney point], is elicited in most cases of acute appendicitis.

"He has occasional palpitations as well as edema after sitting or standing for a prolonged period of time. He also has dyspnea on mild-to-moderate exertion, especially over the last couple of months. He also complains of right calf discomfort when walking for about half a block. He has to stop, and his discomfort is typically relieved after a few minutes. His weight is 220 pounds. Blood pressure of 116/74 mmHg is well controlled on Norvasc. Heart rate is 85 beats per minute. The echocardiogram done today showed good left ventricular systolic function with no segmental wall motion abnormalities or left ventricular hypotrophy. Mild aortic sclerosis was also seen." Which of the following was transcribed incorrectly in the above passage? A palpitations B mmHg C systolic D hypotrophy

D The patient is known to be hypertensive, yet controlled, on Norvasc. Since hypertension is known to enlarge the left ventricle, the physician would be checking for left ventricular hypertrophy, not hypotrophy.

Which of the following is not a bone in the hand? A lunate B triquetrum C pisiform D talus

D The talus is one of the seven tarsal bones found in the ankle. All three of the others [lunate, triquetrium & pisiform] are found in the proximal row of carpal bones in the hand.

The medical term for undescended testicles is: A varicocele B hypospadias C hyperspadias D cryptorchidism

D This is the most common genital problem encountered in pediatrics, occurring in 3% of all newborn males. If untreated, it can have deleterious effects on the testis over time.

According to the Book of Style, what is one of the factors impacting SRT adoption and integration? A. dictation habits B. federal legislation C. type of healthcare facility D. range of medical specialties involved

A

An SRT draft document showing exam findings post spontaneous vaginal delivery contains the sentence, "There were no noted peroneal abrasions or lacerations." The MT editor should: A. Correct "peroneal" to "perineal" and send the report. B. Leave the report as is and send for physician signature. C. Flag for physician to clarify. D. Leave the sentence as is, adding [sic] to indicate verbatim.

A

Choose the correctly transcribed sentence. A. A Deaver retractor was placed and the abdominal cavity was explored. B. A Beaver retractor was placed and the abdominal cavity was explored. C. A Deaver blade was used to create a corneal incision. D. A Deaver blade was used to create an abdominal incision.

A

For placement over the dorsum, a cartilage crusher was used to crush and flatten a small piece of septal cartilage, then the contour was checked and confirmed to be satisfactory. Which surgical procedure would contain the above language? A. rhinoplasty B. tracheotomy C. cervical diskectomy D. myringotomy

A

Given a facility's use of voice recognition technology, which of the following may be the responsibility of the documentation specialist (MT)? A. draft documenting editing B. real-time correction of draft document C. dictation of narrative D. authentication of reports

A

The Centers for Medicare and Medicaid have worked in collaboration with The Joint Commission to develop a requirement that hospitals collect evidence-based data on certain identified conditions and treatment programs. What is this set of required care processes called? A. core measures B. meaningful use C. pay for performance D. data analytics

A

The condition of ascites is the accumulation of fluid where? A. peritoneal cavity B. apex of lungs C. pericardium D. parietal pleura

A

What condition is suggested by high blood pressure, protein in the urine, and seizures? A. eclampsia B. discoid lupus C. Parkinson disease D. diabetes insipidus

A

Which of the following is correctly transcribed? A. A gated scan revealed ostial plaque. B. A gaited scan revealed ostial plaque. C. A gated scan revealed osteal plaque. D. A gaited scan revealed osteal plaque.

A

Which of the following requires the dictator to edit text as dictation occurs? A. front-end speech recognition B. natural language processing C. back-end speech recognition D. all speech recognition engines

A

Which refers to the application of natural language processing using a clinical nomenclature system, such as SNOMED-CT? A. data tagging B. data mining C. data abstracting D. data integration

A

Which sentence correctly expresses a radiation dose level during radiotherapy? A. Starting dose was 900 cGy B. Starting dose was 900 CGY. C. Starting dose was 900 Cgy. D. Starting dose was cgray.

A

Which type of scissors might be used in vascular surgery? A. Potts B. Mayo C. Stevens D. bandage

A

Volkmann and Haversian canals are found in the: A bone B brain C liver D kidneys

A Volkmann canals are also called the perforating canals because they exist to allow blood vessels, lymph vessels and nerves to penetrate compact bone. The Haversian canals provide space for these vessels and nerves to run longitudinally through the central portion of the bone where they diverge through canaliculi and communicate with bone osteocytes for exchange of nutrients/oxygen and waste materials in the bone.

VRE is treated with which of the following? A Zyvox B Zovirax C Zofran D Zoladex

A Zyvox (linezolid) was the first drug approved by the FDA in a new class of drugs designed to treat nosocomial infections such as vancomycin-resistant Enterococcus faecium (VREF) and methicillin-resistant Staphylococcus aureus (MRSA) as well as hospital-acquired pneumonia.

Which refers to the process of gathering and storing information from patient progress notes, imaging reports, transcribed notes, and other clinical sources into a single healthcare record? A. documentation B. data capture C. record keeping D. interoperability

B

Which refers to the transcribed document after the application of NLP? A. data file B. codified narrative C. abstracted dictation D. source file

B

Choose the correct pathway for inhaled air. A. pharynx, trachea, larynx, bronchi, bronchioles, lung capillaries, alveoli B. pharynx, larynx, trachea, bronchi, bronchioles, alveoli, lung capillaries C. larynx, pharynx, trachea, bronchi, bronchioles, alveoli, lung capillaries D. pharynx, larynx, trachea, bronchioles, bronchi, lung capillaries, alveoli

B

DICOM is the interoperatility standard established to support distribution and access to which of these? A. e-prescriptions B. imaging studies C. laboratory data D. home health reports

B

Front-end SRT may include which of the following? A. flagging of report for physician clarification B. physician correction of draft document C. training the MT to edit D. reformatting of reports prior to authentication

B

In neonatology, what does CRL mean? A. chest, ribs, lungs B. crown-rump length C. cranium-rump length D. critical restriction limits

B

In the picture at the right, what anatomical feature is indicated by the arrow? (picture of kidney with two "stalks" coming out next to each other and one stalk hanging down.) A. common bile duct B. ureter C. duodenum D. renal artery

B

What does ther phrase meaningful use refer to? A. Appropriate application of clinical coding standards to avoid fraud and abuse in patient billing. B. Adoption and integration of EHR systems in a manner that drives coordinated use of health information and improved care outcomes. C. Capture of core measures information that will generate useable statistical data to support the management of specific diseases. D. Integration of e-prescribing technologies that eliminate drug-related data entry errors.

B

What is a treatment for cystocele? A. cystectomy B. colporrhaphy C. cecostomy D. chordotomy

B

What is located at the upper left portion of the abdomen and is the largest of the lymphoid organs? A. thymus B. spleen C. liver D. gallbladder

B

What is the name of the sac located on the undersurface of the liver? A. pancreas B. gallbladder C. appendix D. duodenum

B

The piriformis muscle is located A in the shoulder extending to the upper arm. B in the pelvic region extending to the thigh. C in the upper arm extending to the elbow. D next to the frontal sinuses.

B The piriformis muscle originates in the pelvic, passes through the sciatic notch and inserts at the greater trochanter. Its action is to rotate the thigh laterally. The muscle is shaped like a pear, thus the name piriformis.

In the picture at the right, what anatomical feature is the arrow pointing to? (picture of external ear and internal ear. Pointing to several round circle structures in the inner ear) A. incisive canal B. hypoglossal canals C. semicircular canals D. pterygoid canal

C

In which type of surgery might a peel-away sheath be used? A. repair of an AVM B. FNA of the breast C. implantation of an AICD D. implantation of an IOL

C

Natural language processing is also referred to as natural language _____. A. interpretation B. recognition C. understanding D. management

C

Needles were passed with direct palpation through the vaginal incision from the lateral thigh to the vaginal incision. Which operative report would include the above sentence? A. hysterectomy B. cystectomy C. urethral sling D. vulvectomy

C

On an electromyography report, which phrase is correctly expressed? A. Sensory latency 0.1 mls B. Sensory latency 0.1 mS C. Sensory latency 0.1 msec D. Sensory latency 0.1 mSec

C

Pain and paresthesias in the thumb and index finger, especially at night, are symptoms of what condition? A. rheumatoid arthritis B. Colles fracture C. carpal tunnel syndrome D. Dupuytren contracture

C

The SRT draft document repeatedly refers to the patient as Charles Smith in the body of the report. This does not match the ADT feed/demographic header of "Casey Smith." What step should the editor take? A. leave the report as is and send for physician signature B. change "Charles Smith" to "the patient" throughout the report C. flag for physician to clarify D. change demographic header to "Charles Smith"

C

To allow mobilization of the tissue, the galea was incised in multiple areas, then the tissue was closed with 3-0 Vicryl suture deep in the galea. In what kind of surgery would the above sentence be found? A. Total hip replacement B. Total knee replacement C. Scalp tumor resection D. Resection of renal tumor

C

What condition is being described when a patient is classified as Dukes B? A. malignant melanoma B. leukemia C. colon carcinoma D. lymphoma

C

What condition might be present in a patient with an elevated alkaline phosphatase? A. microcytic anemia B. glioblastoma C. biliary obstruction D. diabetes mellitus

C

Which of the following would edit front-end SRT? A. certified medical transcriptionist/editor B. certified physician's assistant C. physician D. QA specialist

C

Which organ is involved in the removal and excretion of metabolic wastes from the blood? A. liver B. spleen C. kidney D. colon

C

Which organization sets standards for interoperability and publishes specifications/protocols used by software developers who create interfaces between health IT computer systems? A. SNOMED B. AMA C. HL7 D. CDA

C

According to the Book of Style, 2nd edition, for which of the following would roman numerals not be used to express a value? A Clark levels B NYHA classification for heart failure C Glasgow coma scale D EKG standard bipolar leads

C The Glasgow coma scale, which describes level of consciousness in patients with head injuries, tests the patient's ability to respond to verbal, motor and sensory stimulation the values of which are expressed on a scale of 1 through 5 and then totaled to indicate level of consciousness. These are documented as Arabic numerals. All three of the others are expressed with roman numerals.

A Mayfield holder would be used in what type of surgery? A. abdominal B. thoracic C. cardiac D. cranial

D

A child diagnosed with a crush injury to the physis would be classified how? A. LeFort I B. Rancho Los Amigos VI C. Mayo IIIA D. Salter-Harris V

D

A patient with a MELD score of 19-24 may be a candidate for what procedure? A. knee replacement B. heart transplant C. cataract surgery D. liver transplant

D

A rural clinic is comparing costs among SRT vendors. Which of the following may influence the actual return on investment? A. number of sales visits per contracted year B. projected cost savings as per vendor C. ROI of large hospitals in the area D. training and implementation costs

D

For what condition is Benicar prescribed? A. hypotension B. type 2 diabetes C. breast cancer D. hypertension

D

The SRT draft document contains the sentence, "The patient is wide court elevator." On reviewing the audio, the MT editor determines the voice file is inaudible. S/he should: A. delete the sentence B. leave the sentence as is, adding [sic] to indicate verbatim C. edit to "The patient's white count was elevated." D. flag for clarification

D

What condition would be treated with a prescription for the topical retinoid tazarotene? A. Crohn disease B. candidiasis C. pediculosis capitis D. psoriasis

D

Which of the following is a speech recognition technology that uses traditionally recorded voice files? A. OAD transcription B. natural language processing C. front-end SRT D. back-end SRT

D

Which phrase correctly uses the medical term? A. actinic cell carcinoma B. actinic ketosis C. acinic keratosis D. actinic keratosis

D

Along with other laboratory results, the dictator says "polys 75." The dictator is referring to: A a liver enzyme. B an amino acid level. C a triglyceride value. D a type of white cell.

D "Polys" is a common short form of polymorphonuclear leukocytes PMNs), the most common white cell type found in the peripheral blood. This particular white cell has a nucleus that has many (poly) shapes (morpho). The nucleus appears pinched or segmented, similar to a string of sausages, hence the other common name "segs."

In neurology, what is an abbreviation for a test that measures stimulated electrical signals? A. BNP B. PGF C. BSER D. BAEP

D brainstem auditory evoked potential

What is the name of the bone to the front of the ear? A. occipital B. parietal C. frontal D. sphenoid

D occipital is back of the head parietal is top of the head frontal is front of the head

What is the correct abbreviation for chronic fatigue and immune dysfunction syndrome? A. CFIDS B. CFAIDS C. CFIMDS D. CFIDSY

A

In the abbreviation ICD-10, what does the "D" refer to? A. disease B. diagnosis C. division D. diagnostics

A

Which term is used to describe a computer system that is old and outdated? A. archive B. legacy C. sunset D. next gen

B

The medial borders of the longus colli muscles were dissected free from their attachments. A needle was placed and an x-ray properly localized the space. In which surgical proceduer would you find the sentences above? A. anterior cervical diskectomy B. Achilles tendon repair C. rhytidectomy D. thyroidectomy

A

What diagnosis does a positive fluorescent ANA suggest? A. presence of lupus B. absence of lupus C. absence of folliculitis D. presence of acute nephritis

A

What does a Rancho Los Amigos score of VIII signify? A. purposeful and appropriate behavior B. inresponsiveness C. confused and agitated behavior D. generalized response to stimuli

A

Calcitonin is produced by which gland? A thyroid B pituitary C adrenal D thymus

A Calcitonin is produced by the parafollicular cells of the thyroid gland. Together with parathyroid hormones and calcitriol, calcitonin maintains homeostasis of calcium and phosphates in the blood. It lowers the amount of blood calcium and phosphates by inhibiting bone breakdown and accelerating uptake of calcium and phosphates by bones.

A patient with nyctalopia suffers from: A night blindness B blurred vision C near-sightedness D double vision

A Nyctalopia, or night blindness, is an inability to see well at low light levels and is most often caused by a prolonged vitamin A deficiency and the consequent inability to synthesize a normal amount of rhodopsin.

Which BUN value is within the normal range?k A. 12 B. 2 C. 122 D. 200

A

The prefix ipsi- means: A same. B opposite. C right. D left.

A Ipsi- is derived from the Latin ipse which means "same." Ipsilateral, meaning the same side, is a common use of this prefix. (The dilated pupil was ipsilateral to the extradural hematoma and contralateral to the paralyzed limbs.)

Jackson-Pratt and Penrose are examples of: A drains B sutures C rongeurs D needles

A Jackson-Pratt and Penrose are examples of surgical drains.

Bruxism is defined as chronic: A teeth grinding B hiccupping C belching D grunting

A Often a precursor to TMJ, requiring the use of a mouth guard, bruxism is the compulsive act of grinding or clenching one's teeth, typically at night. Most patients are not aware of the compulsion but do develop symptoms of headache, pain in the jaw and neck stiffness as a result. Many may also compulsively bite their fingernails, pencils or the insides of the cheek.

The suffix -azine identifies a group of generic drugs used to treat: A psychosis B gouty arthritis C inflammation D viral infection

A The ending -azine is common to generic phenothiazine antipsychotic drugs, such as chlorpromazine [Thorazine], prochlorperazine [Compazine], and trifluoperazine [Stelazine]. In the case of these three, even the brand retained the identifying suffix.

Which of the following is a digestive enzyme? A lipase B bile C albumin D creatine

A Lipase is a digestive enzyme which digests fat. The term is derived from lipos meaning fat and the suffix -ase, denoting an enzyme. Enzymes often have the suffix -ase, which is appended to the name of the substrate (the molecule) that the enzyme acts upon. Other digestive enzymes include amylase (digests starch) and protease (digests protein). Enzymes that were named before the -ase convention was established use the suffix -in (e.g. pepsin, trypsin).

Which of the following is classified as granular? A basophil B lymphocyte C monocyte D thrombocyte

A Of the five types of leukocytes, three are considered granulocytic because they have conspicuous granules in the cytoplasm. Neutrophils, eosinophils and basophils are thus named because of the types of granules seen when using common hematology stains, such as Wright's stain, which include both acidic eosin and basic dye. Monocytes and lymphocytes are considered agranulocytic because they do not have discernible cytoplasmic granules.

Which of the following is considered a manifestation of the same disease process as major depressive disorder? A dysthymia B hypersomnia C anxiety disorder D bipolar illness

A Major depressive disorder, dysthymia, double depression, and some apparently transient dysphorias all are manifestations of the same disease process. Dysthymia is a chronic mood disorder, with a duration of at least 2 years in adults and 1 year in adolescents and children, manifested as depression for most of the day, occurring more days than not, and accompanied by poor appetite or overeating, insomnia or hypersomnia, low energy or fatigue, low self-esteem, poor concentration, difficulty making decisions, and/or feelings of hopelessness.

Which of the following bones does not articulate with any other bone in the axial skeleton? A hyoid B palatine C lacrimal D vomer

A The hyoid bone is suspended from the styloid processes of the temporal bones by ligaments and muscles. It is located in the neck between the mandible and larynx and provides support to the tongue. All the other bones articulate directly with other bones of the face or skull.

The hard palate is composed of the _________ and __________ bones. A maxillae; palatine B vomer; palatine C sphenoid; ethmoid D maxillae; mandible

A The maxillae, or upper jawbone, and the palatine bone together form the upper palate.

The pectineus muscle is found in the: A thigh B forearm C neck D lower back

A The pectineus muscle, whose action is in flexing and adducting the thigh, originates on the superior ramus of the pubis and inserts at the pectineal line of the femur, between the lesser trochanter and linea aspera.

Which of the following is used to describe the incision utilized in a routine cesarean section? A low-transverse B transabdominal C subcostal D periumbilical

A The typical incision for routine C-section, also called a Pfannenstiel incision, is a transverse incision made low to the abdomen just above the pubic area (a.k.a. the "bikini cut").

Pulmonary edema could be treated in what way? A. tracheotomy and steroid administration B. nasal cannula oxygen and afterload reduction C. pneumonectomy and steroid administration D. pulmonary valve repair and preload reduction

B

In what kind of surgical procedure might a Bookwalter retractor be used? A. catheter extraction B. transsphenoidal resection C. total hip replacement D. laparotomy

D

On echocardiogram, what is evaluated using a mean pressure gradient measurement? A. ventricular hypertrophy B. atrial hypertrophy C. coronary artery narrowing D. aortic valve stenosis

D

ROI of technology integration will be successful with results that show: A. 40% savings in transcription production costs. B. Amortization of hardware and software costs. C. Dictators maintaining more than 80% consistency in their speech pattern. D. Measurable productivity gains.

D

Glomerular filtration, tubular reabsorption, and tubular secretion are involved in what physiologic process? A. urine production B. respiration C. metabolic glucose cycle D. production of thyroid hormones

A

On tonometry testing , what is the range of normal values for IOP? A. 10-21 mmHg B. 100-150 mmHG C. 500-700 mmHg D. 1000-1500 mmHg

A

On x-ray, the radiologist reports a widening at the acromioclavicular joint, suggesting the possiblity of _____, and followup films are recommended. Which condition is likely to be named in the blank? A. separation B. carcinoma C. arthritis D. crush injury

A

Seldinger technique would be used in which procedure? A. heart catheterization B. lithotripsy C. stereotactic radiosurgery D. cesarean section

A

Yeasts and molds are a subdivision of a larger group of pathogens called: A Fungi B Viruses C Flagellates D Bacteria

A Fungi are a kingdom of heterotrophic organisms that include mushrooms, yeasts, and molds

What is the procedure for a hysterosalpingogram? A. A tube with a balloon is passed through the cervix into the uterus and dye is injected. B. A tube with a balloon is placed into the uterus through a tiny pelvic incision and dye is injected. C. Radiopaque dye is administered orally and an x-ray view of the pelvis is taken. D. Radiopaque dye is administered and a pelvic ultrasound is performed.

A

What is the significance of a PT value of 25 seconds? A. Prothrombin time is increased. B. Prothrombin time is normal. C. Partial thromboplastin time is increased. D. Partial thromboplastin time is normal.

A

What medium is used in performing a pulmonary ventilation scan? A. radioactive Xe-133 B. iodine C. carbon-14 D. pure oxygen

A

Which gland is located at the back of the third ventricle of the brain that produces the hormone melatonin? A. pineal gland B. pituitary C. thymus D. hypothalamus

A

Which is a delayed data capture method that is not performed in "real time" at the point of care? A. Dictation/transcription B. Point-and-click template systems C. Front-end speech recognition D. Radiofrequency identification

A

Which is a finding in the Bethesda classification system? A. ASCUS B. MEN 2A C. LUAVPG D. CD3d

A

Which is correctly transcribed? A. The pathology report showed a high mitotic rate. B. Pupillometry showed that mitosis occurred. C. The process of miosis produces sperm cells. D. Mitosis may be a side effect of opioid use.

A

Which veins begin near the angle of the mandible and descend through the neck? A. external jugular B. vertebral C. axillary D. cephalic

A

With the patient in the lateral decubitus position and using arthroscopy, sutures were passed through the superior labrum anteriorly and posteriorly. What surgical procedure is being described in the above sentence? A. repair of SLAP lesion B. repair of patellar tendon C. repair of Dupuytren contracture D. repair of Sinding-Larsen-Johansson lesion

A

The abbreviation YAG refers to a/an: A laser B clamp C bone saw D microscope

A The abbreviation YAG [yttrium aluminum garnet] is seen in association with many modern surgical lasers, most commonly linked with holmium [Ho:LAG], neodymium [Nd:YAG], and erbium [Er:YAG]. It is also seen in combination with a manufacturer or product name, as in Lasertek YAG, Lumonics YAG and Carl Zeiss YAG.

Which of the following is correct? A drug-eluting stent B drug-alluding stent C drug-eluding stent D drug-eluting stint

A A drug-eluting stent is placed in a blocked coronary artery to open the vessel and keep it open (patent). Medication to prevent the artery from becoming blocked again (restenosis) is embedded into the stent material and released slowly in a process called elution. Elute means to separate one material from another using a solvent. In this case, the drug is "separated" from the stent by slowly dissolving into the blood (the solvent).

Which of the following is punctuated incorrectly? A recently-completed workup B cul-de-sac C 3- to 4-cm lesion D The patient is well developed and well nourished.

A A hyphen is not used to join a compound modifier that includes an adverb ending in ly. Cul-de-sac is always hyphenated. Well developed and well nourished are not hyphenated when they follow the noun, but are hyphenated when used as a compound modifier preceding the noun (well-developed patient).

Serum levels of which substance are elevated in CHF? A atrial natriuretic peptide (ANP) B homocysteine C troponin I D myoglobin

A ANP is produced and stored in the atrial myocardium. Since increased synthesis occurs in response to distention of the atria and increased angiotensin II, ANP levels would be elevated in CHF and other conditions associated with from increased blood volume. Homocysteine elevation is associated with premature development of atherosclerosis and risk of MI, troponin I is increased in myocardial necrosis and is an indicator of MI, and myoglobulin is elevated in instances of skeletal muscle damage, specifically of the heart in an MI.

Which is used to enhance the T1 and T2 signals of an MRI? A spin echo B attenuation C signal intensity calibration D administration of contrast

A An advantage of using a spin-echo sequence is that it introduces T2 dependence to the signal. Since some tissues and pathologies have similar T1 values but different T2 values, it is advantageous to have an imaging sequence that produces images with T2 dependence, thus resulting in an image with enhanced detail.

Which of the following refers to normal vision? A emmetropia B presbyopia C hyperopia D myopia

A An emmetropic eye is an eye that can sufficiently refract light rays from an object 6 m away to focus a clear image on the retina. All the others are disorders or abnormalities of vision.

Bacteriuria means: A bacteria in the urine B urine with a high pH C urine with a high specific gravity D bacteria in the blood

A Bacteriuria can be broken down into the prefix bacteri- meaning bacteria plus the suffix -uria denoting urine. Bacteria in the blood would be called bacteremia. Urine with a high specific gravity is called baruria (from baro- meaning weight, heavy) and urine with a high (alkaline) pH is alkaluria.

A patient with bronchial asthma might be prescribed: A Vanceril B Mellaril C Flexeril D Plaquenil

A Beclomethasone is a synthetic steroid of the glucocorticoid family. The naturally-occurring glucocorticoid (cortisol or hydrocortisone) is produced in the adrenal glands. Glucocorticoid steroids have potent anti-inflammatory actions.

pCO2 is reported in which of the following units: A mmHg B mm C cm D mL

A Both pO2 and pCO2 are expressions of partial pressure, and pressures are typically measured in millimeters of Mercury.

Which of the following would be reported on a Pap smear? A CIN B DUB C fibroids D DCIS

A CIN is the abbreviation for cervical intraepithelial neoplasia, a precancerous condition of the cervix. The degree of involvement is indicated by grading 1-3. Dictators often pronounce this finding as "sin-1," "sin-2," or "sin-3."

"He has occasional palpitations as well as edema after sitting or standing for a prolonged period of time. He also has dyspnea on mild-to-moderate exertion, especially over the last couple of months. He also complains of right calf discomfort when walking for about half a block. He has to stop, and his discomfort is typically relieved after a few minutes. His weight is 220 pounds. Blood pressure of 116/74 mmHg is well controlled on Norvasc. Heart rate is 85 beats per minute. The echocardiogram done today showed good left ventricular systolic function with no segmental wall motion abnormalities or left ventricular hypotrophy. Mild aortic sclerosis was also seen." Based on the above passage, the assessment might include: A claudication B type 2 diabetes C congestive heart failure D tachycardia

A Clues to indicate claudication include calf discomfort after walking a short distance that is relieved with rest. Congestive heart failure is not as likely as the patient only has edema after sitting or standing. The heart rate of 85 is not tachycardic.

COX-2 inhibitors are used to treat: A rheumatoid arthritis B Helicobactor pylori C hypertension D heart failure

A Designed to selectively inhibit COX-2, an enzyme involved in the inflammation pathway, COX-2 inhibitors are the newest NSAIDS but have received critical scrutiny from the public and the FDA due to potential cardiovascular and gastrointestinal risks associated with the drugs. The COX-2 drugs Vioxx and Bextra have been pulled from the market due to the overall risk-vs-benefit profile of these drugs.

A fracture caused by weakening of bone due to a disease process is called a/an _________ fracture: A stress B systemic C pathologic D idiopathic

A Diseases such as neoplasia, osteomyelitis, osteoporosis and osteomalacia can cause an gradual erosion of bone that weakens it to the point of spontaneous fracture, often without any traumatic correlation. Those fractures are referred to as pathologic, or caused by disease.

Which of the following abbreviations is not on JCAHO's list of dangerous abbreviations? A b.i.d. B q.h.s C cc D q.d.

A Due to the high probability of misinterpretation, q.d., q.h.s., and cc are all included on the Joint Commission's dangerous abbreviations list, with only q.d. on the minimum list -- items that must be included on each accredited organizations "Do not Use" list. The abbreviation b.i.d. is still considered safe and appropriate for use in documentation.

Which of the following is a carbohydrate? A monosaccharide B triglyceride C polypeptide D eicosanoids

A Eicosanoids and triglycerides are both lipids, dipeptides are amino acid pairs necessary in the formation of proteins, and monosaccharides are simple sugars, or carbohydrates.

Which of the following is correct? A Graves disease B Homan sign C anterior Drawer sign D nursemaid's elbow

A Eponyms (diseases named for individuals) no longer carry the possessive style (e.g., Tinel's sign should be transcribed Tinel sign). Graves disease is named for Robert Graves and always includes the final 's.' Homans sign also includes a terminal 's' as part of the eponym. Anterior drawer sign is not an eponym and should not be capitalized or made possessive. Nursemaid's elbow is written with the possessive because it is descriptive, not eponymic.

Which of the following is a phase of gastric secretion that refers to the reflexes initiated by the sensory receptors of sight, smell, taste or thought? A cephalic B gastric C intestinal D emptying

A Even before food enters the stomach, the sight, smell, taste or thought of food initiates this reflex. The cerebral cortex and feeding center in the hypothalamus send impulses to the medulla, which transmits a parasympathetic impulse to the parietal, chief and mucous cells of the gastric glands, producing secretions in the stomach.

In emergency medicine, ABCs refer to which of the following? A airway, breathing, and circulation B alertness, breathing, and circulation C airway, breathing, and consciousness D alertness, breathing, and consciousness

A First-responders, EMTs, paramedics and emergency personnel are trained to immediately evaluate the ABCs of any victim in the field. Any obstruction of the airway, decrease or cessation in breathing, or compromise of circulation indicates an emergent and potentially life-threatening situation that requires immediate intervention and response.

A patient has a fracture in which the radius is bent but not displaced, and the skin is intact. This type of fracture is known as: A Closed, greenstick B Complex, comminuted C Compound, transverse D Open, spiral

A Greenstick fractures are common in young children whose bones are still highly cartilaginous and pliable. Once bone maturity is reached and the cartilage has been replaced by dense bone, greenstick fractures are less often encountered.

Which of the following can be the culprit for impetigo? A S. aureus B HSV C C. albicans D N. gonorrhea

A Impetigo is a skin infection that is caused by either Staphylococcus aureus or Group A streptococcus. It usually affects preschool and school-age children, especially in the summer months.

In Guillain-Barré syndrome, the presenting symptom is usually: A numbness or tingling in the fingers B muscular atrophy C difficulty breathing D difficulty swallowing

A In Guillain-Barré syndrome, symptoms usually start with numbness or tingling in the fingers and toes, followed by muscle weakness in the legs, arms, and other muscles that develops over a period of days to weeks and can progress to complete paralysis. Difficulties in breathing and swallowing can also develop but are not the presenting symptoms.

Which of the following is the causative agent of walking pneumonia? A Mycoplasma pneumoniae B Mycobacteria tuberculosis C HPV D HIV

A Mycoplasma pneumoniae causes primary atypical pneumonia, also known as walking pneumonia. This form of pneumonia is often not severe enough to require bedrest or hospitalization, hence the name. Mycobacteria tuberculosis is the causative agent of TB, or tuberculosis. HPV is the abbreviation for human papillomavirus, a sexually transmitted disease which causes genital warts, and in some cases, cervical cancer. HIV is the abbreviation for human immunodeficiency virus, the causative agent in AIDS.

Which of the following is transcribed correctly? A Nuclear imaging studies showed a large apical perfusion defect consistent with prior myocardial infarction. B Nuclear imaging studies showed a large apical profusion defect consistent with prior myocardial infarction. C Nuclear imaging studies showed a large apical diffusion defect consistent with prior myocardial infarction. D Nuclear imaging studies showed a large apical effusion defect consistent with prior myocardial infarction.

A Perfuse means to force fluid through an organ or tissue. Normally, the cardiac muscle is fully perfused with blood so that all areas of the heart receive oxygen. A nuclear imaging study shows areas within the walls of the heart which are not receiving adequate blood (adequate perfusion) due to either blockage of an artery or previous damage to the muscle.

A stage 4 pressure sore is one that exhibits: A involvement of bone or joint. B blister or break in the skin. C subcutaneous destruction into muscle. D persistent reddening after relief of pressure.

A Pressure sore staging (Barczak) is described as: (1) skin intact but reddened for greater than 1 hour after relief of pressure, (2) blister or other break in dermis with or without infection, (3) subcutaneous destruction into muscle with or without infection, (4) involvement of bone or joint with or without infection. Stages 1 and 2 are treated conservatively. In general, stage 3 and 4 pressure sores require flap reconstruction. A decubitus ulcer is a pressure sore over a bony prominence that results from the patient lying in the recumbent position (from decumbere, "to lie down"), i.e., affecting the sacrum, heel, and occiput. Technically, pressure sores that result from the seated position are not decubitus ulcers.

When the heart suddenly doubles or triples the number of systolic beats per minute, the condition is known as: A paroxysmal atrial tachycardia B mitral stenosis C heart block D sinus bradycardia

A Rapid heartbeat, particularly of sudden onset, is described as tachycardia.

Which of the following best completes this sentence: "History of confusion with severe left internal _______ artery stenosis seen on Doppler exam." A carotid B thoracic C parotid D mammary

A Severe carotid artery stenosis can cause confusion or other neurological symptoms due to decreased oxygen reaching the brain. The carotid artery is commonly assessed using Doppler ultrasound. The internal mammary artery, also known as the internal thoracic artery, is not in a physiological position to cause brain hypoxia leading to confusion. There is not an internal parotid artery.

Place the following in their correct order from outside to inside: 1. lungs 2. pleural cavity 3. visceral pleura 4. parietal pleura A 4, 2, 3, 1 B 3, 2, 4, 1 C 4, 3, 2, 1 D 1, 2, 3, 4

A Similar to the structures encasing the heart, the lungs are protected by a visceral pleura which is adhered to the external surface of the lungs. The next layer is the parietal pleura, and the pleural cavity is the space that exists between these two layers.

Which of the following might be a sequela of stroke? A intention tremor B retention tremor C attention tremor D distention tremor

A Stedman's Medical Dictionary defines an intention tremor as "a tremor that occurs during the performance of precise voluntary movements, caused by disorders of the cerebellum or its connections." The tremor is most notable in the hands and is not present at rest but only when reaching for or grasping an object. Because the disorder originates in the cerebellum, it is sometimes seen following a stroke.

The term "basketing" is most commonly used in reference to treatment of which clinical finding? A ureteral calculi B atherosclerosis C cholecystitis D rotator cuff tear

A Stone basketing is an interventional procedure performed to extract stones, or calculi, from the ureters. This is often done in conjunction with ureteroscopy and ureteral balloon dilatation.

Which is the correct dose of Synthroid? A 25 mcg B 25 g C 25 mg D 0.025 mcg

A Synthroid dosing can be confusing because it may be reported in micrograms or milligrams, although the manufacturer uses microgram units in all their literature. To convert micrograms to milligrams, move the decimal 3 places to the left. Hint: The larger the unit, the smaller the number, so doses reported in milligrams should be decimals.

The Raji cell assay is performed to evaluate: A autoimmune disorders B blood factors C antinuclear antibodies D cold agglutinins

A The Raji cell assay is regarded as a test for the detection and quantification of immune complexes. It is frequently positive in sera from patients with systemic lupus erythematosus (SLE).

What does the "CT" refer to in SNOMED-CT? A. computerized tracking B. clinical technology C. clinical terms D. computerized terminology

C

Which of the following is transcribed correctly? A Her goal in physical therapy is to have decreased pain during ADLs. B Her goal in physical therapy is to have decreased pain during HDLs. C Her goal in physical therapy is to have decreased pain during AVLs. D Her goal of physical therapy is to have decreased pain during ADL's.

A The abbreviation ADL stands for activities of daily living, important markers of rehabilitative medicine. Remember, it is very difficult to hear the difference between A and H and D and V. Per the Book of Style, use a lowercase 's' without an apostrophe to form the plural of uppercase abbreviations.

The arytenoid cartilage is found in the: A larynx B wrist C pharynx D ankle

A The arytenoids cartilage is one of a pair of small pyramidal cartilages that articulate with the cricoid cartilage to which the corresponding vocal ligament and other muscles are attached.

Where is the buccinator muscle found? A cheek B thigh C eyebrow D lower back

A The buccinator muscle originates at the alveolar process of the maxilla and mandible and inserts at the obicularis oris. It is the major cheek muscle that compresses the cheek, as in blowing air out of the mouth, and causes the cheek to cave in, producing the action of sucking.

Which of the following is transcribed correctly? A A CT scan with myelogram was performed. The C6 root sleeves did not fill well. B A CT scan with myelogram was performed. The C6 route sleeves did not fill well. C A CT scan with myelogram was performed. The C6 route sleeves did not feel well. D A CT scan with myelogram was performed. The C6 root sleeves did not feel well.

A The correct spelling denoting the portion of the nerve exiting the spinal column is root. In this study, dye is used to visualize the nerve roots and normally should fill the sheaths of the nerve. The word "feel" is often transcribed erroneously in this context, yet imaging studies cannot show something which is perceived (sounds, feelings, smells).

The suffix -pramine is indicative of which class of pharmaceuticals? A Tricyclic antidepressants B Antihypertensives C Hyperglycemics D Birth control pills

A The tricyclic antidepressants include desipramine, imipramine, clomipramine, and trimipramine. Amitriptyline and nortriptyline are also classified as tricyclic antidepressants but do not contain the suffix -pramine. These were the most commonly prescribed antidepressants until they were supplanted by the selective serotonin reuptake inhibitors (SSRIs). Common brand names of tricyclic antidepressants include Pamelor, Elavil, Tofranil, and Sinequan.

The visceral pleura is the: A membrane lining the surface of the lungs B membrane lining the wall of the thoracic cavity C fluid surrounding the lungs D thinnest portion of the peritoneum

A The visceral and parietal pleura form the lining structures of the lungs and lung cavity. The visceral pleura adheres to the lungs themselves and the parietal pleura to the chest wall, between which is found the lubricating pleural fluid that allows the visceral pleura to slide over the parietal pleura without friction or rub.

Wernicke's area of the cerebrum is also known as the: A auditory association area. B somatosensory association area. C gnostic area. D visual association area.

A There are six association areas of the cerebrum that consist of association tracts. The auditory association area, or Wernicke's area, determines if a sound is speech, music or noise. It also interprets the meaning of speech by translating words into thoughts.

The ilium is: A part of the hip bone B a bone in the foot C part of the intestine D a bone in the skull

A This is an often-confused sound-alike. Ileum, spelled with an e, corresponds to a part of the intestine while the spelling with an i refers to the large, flaring bone of the hip.

Placenta previa is best diagnosed by: A ultrasound B X-ray C MRI D CT scan

A Ultrasound is 95% accurate and is the test of choice for diagnosing placenta previa, a condition in which the placental tissue lies abnormally close to the internal cervical os and can result in placental detachment, maternal hemorrhage, and possibly fetal hemorrhage. MRI has been shown to be as accurate as ultrasound, but there are concerns about the long-term effects of exposure to a static magnetic field and rapidly shifting gradients on fetal development. Plain radiography and CT scan are of no value in diagnosis of placenta previa.

Which of the following is transcribed correctly? A The patient presents with small vesicles on the back of his right hand and vesical hematuria. B The patient presents with small vesicals on the back of his right hand and vesical hematuria. C The patient presents with small vesicles on the back of his right hand and vesicle hematuria. D The patient presents with small vesicals on the back of his right hand and vesicle hematuria.

A Vesicle (with the ending -le) refers to a small, circumscribed, fluid-filled elevation on the skin (blister). Vesical (ending in -al) refers to a bladder, most often the urinary bladder. Vesical hematuria refers to blood in the urine which originated in the bladder.

What is the name of the muscle on the lateral part of the thigh? A. vastus lateralis B. gastrocnemius C. psoas major D. soleus

A gastrocnemius is calf muscle psoas major is a long muscle (pair) that runs from the lumbar column to the pelvis soleus is also a calf muscle

For which medical specialty would the abbreviation SLR be likely to be found in the physical exam? A. neurology B. infectious disease C. obstetrics D. psychiatry

A straight leg raise

What condition is suspected in a patient with a decreased albumin and an increased ALT? A. ketoacidosis B. primary liver disease C. chronic obstructive pulmonary disease D. sclerosing cholangitis

B

What diagnostic test is typically performed with the patient in a sitting position? A. bone densitometry B. electronystagmography C. intravenous pyelogram D. cholangiography

B

What does the P in NLP stand for? A. practitioner B. processing C. planning D. productivity

B

What does the abbreviation PEEP stand for? A. pulmonary end expiratory pressure B. positive end-expiratory pressure C. positive end-expiration pressure D. positive end-expiratory peak

B

What does the value of GFR indicate? A. Progression of a radioisotope through the kidneys B. Rate at which blood is filtered at the glomeruli C. The amount of protein in urine D. The stage of kidney disease

B

A back-end SRT draft document has no blanks when the editor first views it. The next step would be to: A. spellcheck B. listen to the audio file C. verify diagnoses D. send for physician signature

B

A patient with serum calcium of 12 and a phosphate level of 1.8 would likely have which diagnosis? A. myocardial infarction B. primary hyperparathyroidism C. renal failure D. HIV

B

What is the correct expression of a measurement based on the rate of decay of a radioactive substance? A. MCI B. uMCI C. mCi D. MgC

C

In what type of procedure would you find an Angio-Seal used? A. total hip arthroplasty B. cardiac catheterization C. carotid artery stenting D. rhinoplasty

B

Marcaine 2 mL was used on the superficial skin over the A1 pulley. In what surgical procedure would the above phrase be found? A. total knee arthroplasty B. carpal tunnel release C. closed ORIF D. transforaminal neuroplasty

B

Name the bone that is in the lateral lower arm in line with the thumb. A. humerus B. radius C. ulna D. tibia

B

Several weeks following the onset of pancreatitis, the patient developed an accumulation of thick, purulent fluid, as seen on MRI scan, with an elevated white count and was diagnosed with _____, for which surgery was recommended and performed. What was the diagnosis in the above scenario? A. severe biliary stenosis B. pancreatic abscess C. biliary tract carcinoma D. pancreatic carcinoma

B

This 70-year-old male complains of feeling he cannot empty his bladder completely. He has noted _____ urethral discharge. I have discussed with him that this is likely BPH. Which is the most likely word or phrase to fill in the above blank? A. a greenish B. no C. profuse D. some purulent

B

What class of drugs might be used to treat thrombocytopenia? A. alpha blocker B. corticosteroid C. cephalosporin D. vasopressor

B

A LeVeen shunt is used during or for which of the following? A hydrocephalus B chronic ascites C hemodialysis D carotid endarterectomy

B A LeVeen shunt is a peritoneal-venous shunt used in the management of ascites, particularly in cirrhosis. In tense ascites, where medical treatment has been refractory, paracentesis is the course of treatment and LeVeen shunt is used for this purpose.

The stage of grieving in which a patient seeks to postpone death, even for a short time, is: A anger B bargaining C acceptance D denial

B A grieving patient will typically go through all phases of the process from denial to acceptance in dealing with grief.

The term obtunded would be used to refer to a patient's: A weight B mental alertness C agility D vision

B A patient who is described as obtunded will have experienced some cause for mental bluntness or dulled alertness, often by traumatic injury to the head.

Which of the following refers to a type of polyp that arises from the mucous membrane on a stalk? A sessile B pedunculated C fistulated D hyperplastic

B A pedunculated polyp is a benign growth that protrudes from the mucous membrane of the colon and is attached to that membrane by a stalk or peduncle. A sessile polyp is attached to and sits directly on the mucous membrane.

A scintigram is a record of the: A spinal cord after injection of air or radiopaque substance into the subarachnoid space B distribution of a radioactive tracer in a tissue or an organ C blood vessels to diagnose cardiovascular disease D kidneys, ureters and bladder after injection of radiopaque dye

B A scintigram is used for this purpose. The others are myelogram (B), angiogram (C), and pyelogram (D).

Which is performed to monitor the progression of disease as well as the efficacy of treatment in AIDS patients? A cANCA B CD4 C ELISA D CMV

B A severely depressed CD4 count is the single best indicator of opportunistic infection. Increased CD4 lymphocytes in an HIV-positive patient would indicate a therapeutic effect of the antivirals being administered for the disease. Increased CD4 lymphocytes, however, would indicate immune dysfunction, particularly in AIDS.

A stereotaxic frame is used in: A Arthroscopy B Neurosurgery C Cardiovascular surgery D Obstetrics

B A stereotaxic frame is drilled and affixed to the skull to facilitate radiologic mapping of brain regions for the purpose of diagnosis as well as management of abnormal movements through the introduction of electrodes (also referred to as "stereotactic" frame).

The following were taken from pelvic ultrasound reports. Which one is transcribed correctly? A Left adnexa is normal. B Left adnexa are surgically absent. C Right adnexum are normal. D Right and left adnexum are normal.

B Adnexa (singular adnexum) are accessory structures, and in the case of uterine adnexa, include the fallopian tube, ovary, and associated ligaments. Since the uterus has several accessory structures on each side, the term is always used in the plural, even when referencing only one side, right or left.

Astrocytomas are found in the: A liver B brain C kidney D bone

B An astrocytoma is the most common form of glioma that forms from a star-shaped cell called an astrocyte. These tumors can occur anywhere in the brain and occasionally in the spinal cord, but are typically found in the cerebrum. The most severe, malignant form is glioblastoma multiforme.

The most common type of exocrine gland is: A apocrine B merocrine C endocrine D holocrine

B Apocrine secretion involves secretion or shedding of only the apical portion of a secretory cell. Holocrine secretion involves secretion or shedding of the entire cell. It is merocrine secretion, where the cell fuses with the plasma membrane and releases its contents to the exterior, that is the most common.

Aprosopia refers to congenital absence of the: A eyelashes B face C sexual organs D fingernails

B Aprosopia can be characterized by absence of all or part of the face. It is congenital and very rare, but when it exists, it usually exists along with other physical malformations.

Which of the following would not be prescribed for chronic back pain? A TENS B Bactrim C physical therapy D ESI

B Bactrim is an antibiotic, which would not be used to treat chronic back pain. TENS is an acronym for transcutaneous electrical nerve stimulation. ESI is the abbreviation for epidural steroid injection.

Which of the following would be prescribed for an asthma patient? A proton pump inhibitor B beta agonist C calcium channel blocker D ACE inhibitor

B Beta agonists relax smooth muscle in the airways. Proton pump inhibitors are prescribed for ulcers and GERD to inhibit the production of gastric acid; calcium channel blockers and ACE inhibitors are both prescribed for high blood pressure.

A cachectic patient is: A well nourished. B extremely underweight. C overweight. D jaundiced.

B Cachexia (pronounced ka-kex-ia) refers to a general weight loss and wasting associated with chronic disease or emotional disturbance. This is a common finding in end-stage cancer patients.

The "L" in the abbreviation LEEP stands for: A laser B loop C lysis D local

B LEEP, or loop electrosurgical excision procedure, is used to treat precancerous lesions of the cervix, whereby radio waves are conducted through a wire loop to excise cervical tissue.

The Lee-White method refers to the test for: A platelet function B clotting time C bleeding time D tourniquet time

B Clotting time is the time required for a specimen of whole blood, obtained by venipuncture, to coagulate. The standard method for measuring this is the Lee-White method. For bleeding time, the standard testing is performed either via the Duke method or the Ivy method. The tourniquet test, or Rumpel-Leede test, assess capillary fragility and platelet function. Platelet function can also be assessed via platelet aggregation testing.

Coccidioidomycosis is a _________ disease. A viral B fungal C bacterial D parasitic

B Coccidioides immitis is the fungal culprit that leads to two forms of this disease. The first is self-limited in scope and consists of a respiratory

The suffix -azole is indicative of which class of pharmaceuticals? A antihypertensives B antifungals C NSAIDs D statins

B Common antifungals from the -azole class include ketoconazole (Nizoral), itraconazole (Sporanox), and fluconazole (Diflucan). All of these agents block the production of ergosterol, a crucial component of fungal cell walls. Human cells do not require ergosterol, so the antifungal specifically targets the fungal cells and not the patient's.

Which of the following elements found in the human body is considered a trace element? A phosphorus B copper C iron D magnesium

B Copper is present in the body in such minute concentrations that it is considered a trace element. All trace elements together compose about 0.1% of total body mass. Phosphorus, iron, and magnesium are part of a group of elements that together compose approximately 3.9% of total body mass. It is oxygen, carbon, hydrogen, and nitrogen that make up the difference, composing 96% of total body mass.

Which of the following lab values would be consistent with renal failure? A Hemoglobin A1c 4.9 B Creatinine 2.0, BUN 49 C ALT 100, AST 120 D BNP 20

B Creatinine and BUN are metabolic breakdown products that are eliminated from the body by way of the kidneys. Because they are only excreted through the kidneys, they are ideal for monitoring kidney function. Impaired kidney function reduces clearance of creatinine and BUN; therefore, these levels elevate in the blood. Normal creatinine levels in the blood are quite low, with a normal range of 0.6 to 1.2 mg/dL, and normal BUN values are 5 to 20 mg/dL. Be careful not to confuse creatinine with creatine, which is utilized by muscles for energy.

Which of the following cellular components is not an organelle? A endoplasmic reticulum B cytosol C nucleus D mitochondria

B Cytosol is the semifluid portion of the cytoplasm in which organelles and inclusions are suspended. All the rest [endoplasmic reticulum, nucleus & mitochondria] are organelles, or little organs, each with a highly specialized function within the cell.

Fundal pressure is applied to assist in which of the following? A hemostatic control of a vascular injury B vaginal delivery of a fetus C realignment of displaced bone D immobilization of the head and C-spine

B External pressure is sometimes applied to the top of the uterus when the laboring woman has lost her ability to bear down effectively, though the technique has been associated with controversy related to fetal injury, particularly shoulder dystocia.

The percentage of healthy adults in the U.S. experiencing symptomatic gastroesophageal reflux disease (GERD) each month is: A 40 to 50 B 25 to 40 C 7 to 10 D 10 to 25

B GERD occurs when the amount of gastric juice that refluxes into the esophagus exceeds the normal limit, causing symptoms with or without associated esophagitis. A recent study estimated that 25% to 40% of healthy adult Americans experience symptomatic GERD, most commonly manifested clinically by pyrosis (heartburn), at least once a month. Approximately 7% to 10% of the adult population in the United States experiences such symptoms on a daily basis.

The physician dictates "ach flu." This slang term refers to: A a viral syndrome. B Haemophilus influenza C Asian influenza D Hong Kong influenza

B H flu is the commonly used abbreviated or slang form for Haemophilus influenza, a gram-negative coccobacilli which causes meningitis, conjunctivitis (pink eye) and otitis in children. In adults, it typically causes sinusitis and bronchitis. It was once thought to cause influenza, hence its name. The Hib vaccine is now given to infants and children, primarily to prevent bacterial meningitis caused by H influenza serotype B.

Which of the following terms corresponds to the word parts (blood) + (chloride) + (low, decreased)? A hypochloruria B hypochloremia C hypochlorhydria D hypochondria

B Hypochloremia, built from the prefix hypo- (low, decreased), the suffix -emia (blood), and the root word chlor (chloride), means decreased chloride ions circulating in the peripheral blood. The suffix -uria, as in hypochloruria, would refer to decreased levels of chloride in the urine. Chlorhydria refers to the acid formed by chloride and hydrogen known as hydrochloric acid. Hypochlorhydria is reduced acid in the stomach.

Hyperventilation, as in oxygen deficiency, pulmonary disease or CVA, can result in: A respiratory acidosis B respiratory alkalosis C metabolic acidosis D metabolic alkalosis

B In respiratory alkalosis pCO2 of arterial blood is decreased (below 35 mmHg). Hyperventilation causes the pH to increase. The kidneys will attempt to compensate, but often having the patient breathe into a paper bag and then rebreathe the exhaled mixture of CO2 and oxygen from the bag.

Which of the following are round bacteria that grow in pairs? A streptococci B diplococci C bacilli D staphylococci

B In this bacterial strain, two spherical cells (cocci) join to form a pair like a dumbbell or a figure-of-8. Streptococci, conversely, are formed from cocci chains and staphylococci are formed from cocci clusters. Bacilli are rod-shaped and not part of the cocci group.

Decreased cerebrospinal fluid (CSF) pressure may occur as a result of: A hemorrhage in the brain B blocked spinal canal C swelling in the brain D meningitis

B Increased CSF pressure may occur as a result of swelling or bleeding in the brain, infection (such as meningitis), stroke, or other circulatory problems. Decreased pressure may indicate a blocked spinal canal.

Which of the following lies fully ipsilateral to the left iliac region? A epigastric region B left hypochondriac region C right inguinal region D hypogastric region

B Ipsilateral refers to being on the same side, so in this case, the only region that would be found on the same side (left) would be the left hypochondriac region.

What is the significance of a PTH level of 20 pg/mL? A. It is mildly elevated. B. It is decreased. C. It is normal. D. It is extremely elevated.

C

Patients suffering from bronchospasm are often given mast cell inhibitors (like Intal) to: A suppress the inflammatory response of the immune system B prevent the release of histamine C relax smooth muscle surrounding the bronchi D block the action of leukotrienes

B Mast cells, which are contained in connective tissue and are abundant alongside blood vessels, produce histamine and heparin. Mast cell inhibitors prevent the release of heparin during inflammation. These drugs are not used in treating acute attacks but are given for prophylactic effect. Corticosteroids suppress the inflammatory response, bronchodilators relax smooth bronchial muscle, and leukotriene receptor antagonists block the action of leukotrienes.

The sensation of taste is also known as the ____________ sensation. A olfactory B gustatory C deglutitory D masticatory

B Olfactory sensations relate to smell and gustatory to the sensation of taste. Deglutitory refers to swallowing, of course, and masticatory to the act of chewing.

Which of the following instruments is used for cutting bone? A rasp B osteotome C planer D fixator

B Only an osteotome is designed to actually cut bone. Rasps and planers are designed to shave or file the surface of the bone to desired smoothness. Forceps are for grasping.

The hollow interior of a tubular structure (such as the trachea or colon) is called the: A mucosa B lumen C intima D tunica

B Only the lumen represents the hollow space created by a tubular organ. The terms mucosa, intima and tunica represent layers of organ structures and not the space created by them.

Which condition, in most cases, is the residual of acute rheumatic fever? A patent ductus arteriosis B mitral stenosis C ventricular septal defect D mitral valve prolapse

B Patent ductus arteriosus and ventricular septal defect are both congenital heart defects that are present at birth. Mitral valve prolapse is generally considered an inherited autosomal dominant trait. Mitral stenosis (and sometimes aortic stenosis) occurs most often as a result of rheumatic fever, though symptoms do not usually present until the 30s or 40s.

Anemia that is due to deficiency of vitamin B12 absorption is called: A aplastic B pernicious C hemolytic D erythroblastosis fetalis

B Pernicious anemia is an inherited autoimmune disorder characterized by a lack of secretion of intrinsic factor in the gastric mucosa. The result is a failure to absorb vitamin B12.

Which of the following is a rare, progressive genetic disorder seen primarily in males in early childhood? A keratoacanthoma B porokeratosis C erythrasma D punctuate keratoderma

B Porokeratosis, or keratoatrophoderma, is an autosomal dominant disorder characterized by the formation of slightly atrophic patches surrounded by an elevated keratotic border.

Pain originating from the root of a nerve is referred to as: A reticular B radicular C articular D lenticular

B Radicular comes from the Latin term radix meaning root, as does the vegetable radish. Articular refers to a joint and lenticular refers to a lens (from the Latin word lenticula, a lentil). Reticular refers to a net or mesh. Reticulocytes, immature red cells, derive their name from the root word reticulum because of their characteristic mesh-like staining pattern.

In alternative medicine, Reiki therapy involves the transfer of energy from the provider to the patient without: A being in the same room B touching the patient C any prior knowledge D the use of meditating oils

B Reiki therapy is an Eastern touch therapy in which the practitioner systematically uses light hand placement in one of 12 positions on the recipient's body to balance and direct healing energy to those sites

Which of the following is prescribed for sleep? A Celebrex B Rozerem C Xalatan D Remeron

B Rozerem (ramelteon) is a new class of nonnarcotic sleep aid which is reported to not cause dependence. Remeron is an antidepressant, Celebrex is an antiinflammatory, and Xalatan is used to treat glaucoma.

Where are Schwann cells found? A squamous epithelium B peripheral nerve axons C myocardial tissue D glomerular mucosa

B Schwann cells, also called neurolemmocytes, are flattened cells arranged around the axons of nerves in the PNS (peripheral nervous system). They produce myelin sheath around axons of PNS neurons.

A sialolith is what kind of stone? A pancreatic B salivary C kidney D gall

B Sialoliths are calcareous concretions or calculi in the salivary ducts or glands, involving most commonly the submaxillary gland and its duct, less frequently the parotid and sublingual glands and their ducts, and seldom the minor salivary glands.

Which of the following might result in an elevated urine specific gravity? A proteinuria B diabetes mellitus C diabetes insipidus D hemoglobinuria

B Specific gravity is increased in conditions that result in an increased concentration of dissolved materials in the urine. Normally, plasma glucose is reabsorbed in the renal tubules. In the case of diabetes mellitus, however, the renal threshold for glucose is surpassed at about 180 mg/dL, and the spill over into the urine will result in an increased specific gravity.

Which of the following is not a term used to describe the shape of a cell? A squamous B stratified C columnar D cuboidal

B Squamous cells are flattened and scalelike, cuboidal cells are usually cube-shaped in cross section, and columnar cells are tall and cylindrical or somewhat rectangular. The term stratified refers to epithelium composed of stacked layers of cells, typical in areas of the body with a high degree of wear and tear. This has nothing to do with the shape of the cells themselves.

Barrett esophagus is a possible complication in patients with what diagnosis? A. TMJ B. GERD C. EBV D. ESRD

B Temporomandibular joint Epstein-Barr virus end stage renal disease

The class of drugs commonly referred to as statins are used to treat: A bacterial infections. B dyslipidemia. C hypertension. D acne.

B The "statin" drugs include lovastatin (Mevacor), atorvastatin (Lipitor), simvastatin (Zocor), pravastatin (Pravachol) and rosuvastatin (Crestor). These medications are used to treat dyslipidemia by reducing LDL cholesterol, the form of cholesterol considered to be most harmful to arteries.

What procedure inserts a catheter percutaneously and uses an attached balloon to crush an obstruction? A. DSA B. IVP C. PTCA D. PTHC

C

Which of the following scales is used to assess the physical status of a newborn infant? A Karnofsky B Apgar C APGAR D Cattell

B The Apgar scale, named for Virginia Apgar, MD, is used to assess the physical status of a newborn infant, but this term is often confused with the acronym APGAR (adaptation, partnership, growth, affection, resolve), which is a family assessment instrument. The Karnofsky scale is used to assess a person's activity level after a therapeutic procedure, especially chemotherapy. The Cattell Infant Intelligence Scale is used to assess the cognitive development of infants 3 to 30 months of age.

Which of the following tests uses a Bruce protocol? A Angiogram B Exercise stress test C Color Doppler D MRI of the brain

B The Bruce Protocol is used during an electrocardiogram-monitored exercise stress test to outline the progression of speed and elevation of the treadmill to create an increasing exercise load.

The Joint Commission (JCAHO) grants accreditation for a maximum of: A Five years B Three years C One year D Eighteen months

B The Joint Commission on Accreditation of Healthcare Organizations (JCAHO) evaluates and accredits more than 15,000 healthcare organizations and programs in the United States. The review is used internally by healthcare organizations to support performance and externally to demonstrate accountability to the public. Accreditation is on a 3-year review cycle.

Which of the following is transcribed correctly? A A wide S wave is seen in leads 2, V5, and V6. The QRS is also prolonged as in right bundle branch block. B A wide S wave is seen in leads II, V5, and V6. The QRS is also prolonged as in right bundle branch block. C A wide S-wave is seen in leads II, V5, and V6. The QRS is also prolonged as in right bundled branch lock. D A wide S wave is seen in leads II, V-5, and V-6. The QRS is also prolonged as in right bundle branch block.

B The bipolar leads are designated with roman numerals (I, II, and III). Precordial leads are written with a capital V and arabic numbers (V1, V2, V3). Subscripts may also be used (V1, V2, V3), but hyphens are not used to designate any of the 12 leads used in an EKG. Answer C is incorrect because "S wave" is not a compound modifier and does not need to be hyphenated. You would use a hyphen if the phrase was used as a compound modifier such as "S-wave abnormality."

Which of the following is transcribed correctly? A Return to the clinic for further blood tests to look for rheumatoid factors and an ESR or nonspecific measure of inflammation. B Return to the clinic for further blood tests to look for rheumatoid factors and an ESR, or nonspecific measure of inflammation. C Return to the clinic for further blood tests to look for rheumatoid factors and an EFR or nonspecific measure of inflammation. D Return to the clinic for further blood tests to look for rheumatoid factors and an EFR, or nonspecific measure of inflammation.

B The correct abbreviation is ESR (erythrocyte sedimentation rate), not EFR. The phrase "or nonspecific measure of inflammation" is an appositive, not a separate test, and should be set off by a comma. Appositives identify or explain the nouns or pronouns which they modify.

Humans have two sets of teeth. The first set, the baby teeth, are also called the ___________ teeth. A secondary B deciduous C permanent D dentitious

B The deciduous, or primary, teeth begin to erupt at 6 months of age and one pair appears at about each month thereafter until all 20 are present. All of the deciduous teeth are lost between 6 and 12 years of age and are replaced by the permanent, or secondary, teeth.

A tort is a: A criminal wrong B civil wrong C misdemeanor D willful, premeditated act

B The distracters to this question all imply criminal behavior for which prosecution would be pursued by legal authorities. A tort is a negligent or intentional civil wrong causing injury to someone, for which the injured party can sue the wrongdoer for damages. This is significant to medical practice because it is through the tort system that most medical malpractice claims are pursued.

The great saphenous vein runs from: A shoulder to elbow B hip to dorsal foot C knee to dorsal foot D neck to shoulder

B The great saphenous vein, the longest in the body, begins at the medial end of the dorsal venous arch of the foot. It empties into the femoral vein in the groin. The cephalic veins run shoulder to elbow, the small saphenous veins from posterior knee to dorsal foot, and the subclavian from neck to shoulder.

Which of the following controls temperature, sleep and appetite? A adrenal glands B hypothalamus C pancreas D thyroid

B The hypothalamus is a small portion of the diencephalon located below the thalamus. It is divided into a dozen or so nuclei in four major regions that control many body activities. The hypothalamus is one of the major regulators of homeostasis.

The position in which the patient lies supine with the thighs abducted and the feet supported in stirrups is called the: A Trendelenburg position B lithotomy position C Sims position D lateral decubitus position

B The lithotomy position, common to gynecological exams and surgical procedures, involves the patient lying in the supine position with feet elevated and supported in stirrups.

Which word would describe the "vehicle" by which an infectious agent is transferred from an infected to a susceptible host? A endemic B vector C pathogen D pandemic

B The organism that hosts the pathogen and then transmits it to a susceptible host is known as the vector. Mosquitoes, for example, are the vectors for many blood-borne diseases.

The normal value range for albumin is: A 15-45 mcg/dL B 3.5-5.0 g/dL C 24-30 mEq/L D 5-20 mg/dL

B The other value ranges provided are for: (a) ammonia, (c) bicarbonate, and (d) BUN.

The scalene muscles are located in the: A back B neck C abdomen D forehead

B The scalene muscles refer to any of three muscles on each side of the neck that serve to bend and rotate the neck and that assist breathing by raising or fixing the first two ribs. Also called scalenus.

The suffix -profen is indicative of which class of pharmaceuticals? A antibiotics B NSAIDs C antihistamines D sleep aids

B The suffix -profen is used in the common nonsteroidal inflammatory medications ibuprofen, ketoprofen, and also in the narcotic-NSAID combination Vicoprofen.

Korotkoff sounds are evaluated when: A counting the apical heartbeat B determining the blood pressure C performing ultrasonography D counting the respirations

B The various sounds that are heard while taking blood pressure are called Korotkoff sounds. The first Korotkoff sound (the first sound heard) corresponds to systolic blood pressure, and the fifth Korotkoff sound (when sound disappears) corresponds to diastolic blood pressure.

The measurable quantity of a virus in a patient's blood stream is known as the viral _____________: A volume B load C quotient D ratio

B The viral load speaks to the volume of genetic viral material contained in the bloodstream at any given time. HIV and hepatitis C are examples of viral diseases that require monitoring of a patient's viral load for the purpose of assessing the course or status of the disease.

The greater and lesser wings are associated with which structure of the skull? A occipital bone B sphenoid bone C ethmoid bone D parietal bone

B This bat-shaped structure lies at the middle part of the base of the skull and is called the keystone of the cranial floor because it articulates with all other cranial bones, holding them together.

Pityriasis refers to a/an: A inflammation of the urethra B seasonal skin rash C bacterial infection of the conjunctiva D fungal infection of the nailbeds

B This disorder is a common skin rash of young people, especially young adults. It occurs most commonly in the fall and spring, and a virus is suspected as the cause. Although pityriasis rosea may occur in more than one person in a household at a time, it is not thought to be highly contagious. Attacks generally last 4 - 8 weeks. Symptoms may disappear by 3 weeks or last as long as 12 weeks.

Which of the following bandages is used to exsanguinate a limb? A Kling B Esmarch C Kerlix D Telfa

B This sterile rubber bandage is applied around a part from distal to proximal in order to expel blood from it. The limb is often elevated as this pressure dressing is applied.

A tumor or tissue that is chopped into small pieces in order to be removed is said to be ____________. A dessicated B morcellated C lysed D masticated

B Tumors, tissue, and even organs are morcellated for the purpose of removing them via laparoscopic techniques in small manageable and extractable pieces.

CIN, or cervical intraepithelial neoplasia, is expressed in grades from 1 to: A 2 B 3 C 6 D 10

B Used to delineate preinvasive cancerous lesions of the cervix, the designation measures the degree of dysplasia from 1 (early) to 3 (carcinoma in situ).

Which of the following is a protein associated with iron transport? A ceruloplasmin B transferrin C ferritin D Alpha1 antitrypsin

B While ferritin is associated with iron and can be an indicator of chronic iron deficiency, it is not responsible for transport of iron. Ferritin is stored in chemical combination with iron, but transferrin is actually responsible for transporting iron in the blood stream.

What muscle is one of the largest in the back? A. infraspinatus B. latissimus dorsi C. adductor magnus D. bluteus medius

B infraspinatus is small muscle of back overlying scapula. adductor magnus is muscle on inner thigh gluteus medius is hip muscle

What kind of diagnostic test is performed when an opaque dye is injected into the salivary glands? A. barium swallow B. lymphangiography C. sialography D. dacryocystography

C

What is the correct meaning of TIMI? A. thrombosis in myocardial infarction B. thrombin inhibition in myocardial infarction C. thrombolysis in myocardial infarction D. transmural inferior myocardial infarction

C

What describes part of the process in which blood glucose levels are regulated? A. The anterior lobe of the pituitary produces insulin and the posterior lobe produces glucagon. B. The adrenal medulla secrets epinephrine and norepinephrine. C. The pancreatic islets produce insulin and glucagon. D. The pancreatic islets secrete epinephrine and norepinephrine.

C

What contrast medium is used in MRI studies? A. radionuclide B. technetium C. gadolinium D. thallium

C

Casodex might be used to treat what disease? A. prostatitis B. urinary tract infection C. metastatic prostate cancer D. epididymitis

C

The term half-life refers to the: A contraindications of a drug. B duration of efficacy for a drug before it expires and must be discarded. C duration of efficacy for a drug within the blood stream before it must be readministered. D decreased susceptibility to the effects of a drug in a patient who has developed a tolerance for it.

C A drug's half-life is the time required for the level of that drug in serum to decrease from 100% to 50%; thus, it is relates to how long the drug remains effective in the blood stream. A drug with a shorter half life needs to be administered more often or manufactured in an extended- or time-released form.

"The patient still complains of a painful sprained ankle. To evaluate for ischemic heart disease, a ___________ nuclear stress test will be done using adenosine." Which of the following would complete the above sentence? A dobutamine B exercise C pharmacological D treadmill

C A pharmacologic agent (adenosine) will be used to stress the cardiac muscle to simulate exercise, since the patient is unable to walk a treadmill or ride a stationary bike on a sprained ankle. Dobutamine is also used to pharmacologically stress the heart, but it would not be used at the same time as adenosine.

Transaminitis, a possible side effect of statin therapy, is indicated by: A decreased hemoglobin. B elevated sedimentation rate. C elevated ALT and AST. D elevated WBC

C ALT (alanine aminotransferase) and AST (aspartate aminotransferase) are enzymes used to transfer amino acids from one molecule to another in key metabolic processes. Elevation of these enzymes may be referred to as "transaminitis."

A short-acting barbiturate given intravenously for the induction of general anesthesia is: A Marcaine B nitrous oxide C Brevital D Versed

C All of the drugs listed are anesthetics. Marcaine is an agent used for local, nerve block or spinal anesthesia. Nitrous oxide, Brevital and Versed are all used for the induction of general anesthesia, but only Brevital and Versed are given IV, and of these, only Brevital is a barbiturate.

Which of the following is not an important role of the liver? A phagocytosis B glycogen storage C excretion of urea D production of blood proteins

C All of these except excretion of urea, which is the role of the kidneys, are vital functions of the liver.

A double-bone fracture is a common term for which of these two bones? A tibia and fibula B maxilla and mandible C radius and ulna D sternum and clavicle

C Also referred to as a both-bone fracture, fractures sustained at the distal ends of both the radius and ulna are typically caused by the patient extending the hand to stop a fall. When a fracture occurs in both bones, the arm is placed in a series of casts beginning above the elbow to prevent rotational displacement of the fractures.

An Esmarch bandage is used to: A stabilize a fractured bone. B cover a burn wound. C exsanguinate a limb before surgery. D wrap the elbow.

C An Esmarch (es´ mark) bandage is used as a tourniquet before and during surgery. The bandage is wrapped firmly to apply pressure to the limb in order to prevent blood flow into the distal limb.

Which of the following is transcribed correctly? A Dr. Smith will give a lecture on Cardiology in the auditorium today. B We will wait for the Pathology report. C She works in Microbiology. D The patient will be referred to Dr. Jones, an Orthopedist.

C As per The AAMT Book of Style, 2nd Edition, page 47, capitalize department names when referred to as an entity. Use lowercase for common nouns designating a department name. Medical specialties are also not capitalized (see page 250).

Which of these presenting symptoms might result in a BNP being ordered? A pyuria B hematemesis C shortness of breath D epistaxis

C BNP refers to brain natriuretic peptide, a substance whose elevated plasma levels are indicative of congestive heart failure (CHF), of which shortness of breath is a common presenting symptom.

A patient's lab values are reported as sodium 125, potassium 3.0, BUN 30, creatinine 4.0, troponin I normal at 1.0, and CK normal at 35. Based on these values, which of the following could be included in the assessment? A Status post myocardial infarction B Hyperkalemia C Renal disease D Diabetes mellitus

C BUN and creatinine are byproducts of metabolism that are normally found in the urine. Because there is no other mechanism for excreting BUN and creatinine, elevated values often indicate renal disease. Sodium and potassium values are also abnormal, which can also indicate kidney disease. A myocardial infarction would not be indicated, since both Troponin I and CK are normal. The potassium value of 3.0 is low, indicating hypokalemia, not hyperkalemia.

Candida albicans is best treated with: A Savlon B Daktarin C Clotrimazole cream D metronidazole gel

C Candida albicans is yeast that necessitates treatment with an antifungal medication, such as clotrimazole. This active ingredient interferes with fungal cell membranes and prohibits the production of ergosterol, essential to the stability of the cell membrane.

Necrosis of tissue resulting in the conversion of damaged tissue into a soft, cheesy substance is called: A decomposition B necrotization C caseation D fermentation

C Caseating tissue is seen in some granulomas of lung tissue, where the inflammatory cells (granuloma) are dry and crumbling, like a soft cheese.

Ménière syndrome is characterized by a gradual loss of what function? A gait B vision C hearing D swallowing

C Characterized by an increased amount of endolymph that enlarges the membranous labyrinth of the ear, Ménière syndrome can result in fluctuating hearing loss (caused by distortion of the basilar membrane of the cochlea), attacks of vertigo, and roaring tinnitus. Over a period of years, there may be almost total destruction of hearing.

The technique for administering drugs that follow the body's biorhythms is called: A biotherapy B aurotherapy C chronotherapy D balneotherapy

C Chronotherapy [chronos - time] has been proven very effective, particularly in the treatment of asthma, arthritis and cancer, where coordination of drug administration to the body's natural biorhythms has proven more effective than traditional administration. The other therapies [bio - treatment with antibodies, auro - treatment by injection of gold salts, & balneo - treatment with baths] do not deal with body rhythms.

It is acceptable to use a hyphen between the limits of a range if: A The phrase "from...to" is used. B Both values are negative. C Neither value is accompanied by a symbol. D Both values contain decimals and/or commas.

C See page 347 of the AAMT Book of Style, 2nd edition, for the five conditions that must be met in order to use a hyphen between the limits of a range.

Coccidioidomycosis is sometimes referred to as: A catscratch disease B Legionnaires disease C San Joaquin Valley fever D tularemia

C Coccidioidomycosis is caused by Coccidioides immitis, a dimorphic soil fungus native to the San Joaquin Valley of California, southern portions of Arizona, northern portions of Mexico, and scattered areas in Central America and South America. It is sometimes referred to as San Joaquin Valley fever. Legionnaires disease is caused by the Legionella pneumophila organism. Catscratch disease is a benign adenopathy usually caused by Bartonella henselae. Tularemia is a tick-borne disease originating in Tulare, also in the San Joaquin Valley, caused by Francisella tularensis.

Debridement consists of: A bringing raw wound edges into alignment B occlusion of open and bleeding vessels C removal of infected and devitalized tissue D reopening of an improperly healed wound

C Debridement of tissue involves cleaning an area, typically a wound or incision, of dead and dying tissue. The term is often used in conjunction with the word "irrigation," as in "irrigation and debridement" (also known as I&D, but not to be confused with "incision and drainage").

CD rods are used to treat: A comminuted femoral fracture B kyphosis C scoliosis D chondromalacia patellae

C Drs. Cotrel and Dubousset designed this popular form of spinal instrumentation, which was the first type of rod system to use multiple hooks on one rod that could be turned in either direction.

In ophthalmology, ductions are eye movements that are: A conjugate B disconjugate C monocular D binocular

C Ductions are monocular eye movements. Movement of the eye nasally is adduction; temporal movement is abduction. Elevation and depression of the eye are termed sursumduction (supraduction) and deorsumduction (infraduction), respectively. Incycloduction (intorsion) is nasal rotation of the vertical meridian; excycloduction (extorsion) is temporal rotation of the vertical meridian.

Which type of blood cell proliferates in the presence of a parasitic infection or allergic response? A basophils B neutrophils C eosinophils D monocytes

C Eosinophils will leave the capillaries and enter tissue fluid where they are believed to release enzymes, such as histaminase, that combat the effects of histamine and other mediators of inflammation in allergic reactions. They also phagocytize antigen-antibody complexes and are effective against certain parasitic worms.

The Breslow index is used to classify which of the following? A liver tumors B heart failure C melanoma D cervical neoplasia

C Established in 1970, this pathological staging system for melanoma is based on measurement of tumor invasion of the dermis using the micrometer on the microscope. The actual measurement of the depth of the lesion is recorded.

Which of the following is the percentage of the volume of a blood sample represented by red blood cells? A differential B hemogram C hematocrit D hemoglobin

C Hematocrit is defined as the percentage of blood volume represented by the red blood cells. Hematocrit comes from the two combining forms hemato, meaning blood, and krinos, meaning to separate. Hematocrit was originally measured by centrifuging a blood sample (which separates the cells from the watery portion) and comparing the height of the column of red cells to the height of the column of red cells and plasma combined. In modern laboratories, the hematocrit is calculated using automated equipment, although using a centrifuge remains the "gold standard."

Hallucinations that occur during the transitional state at the onset of sleep are called: A somnolent B apneic C hypnogogic D soporific

C Hypnagogic hallucinations occur most frequently just at the onset of sleep when the mind and body are transitioning to a sleeping state. Similar hallucinations can occur during the same transition period of waking from sleep, referred to as hypnapagogic.

On an electrocardiogram, atrial depolarization is represented by which of the following? A ST segment B QRS complex C P wave D T wave

C In a typical Lead II record (right arm to left leg), three clearly recognizable waves accompany each heartbeat. The first is the P wave that represents atrial depolarization. The QRS complex represents the onset of ventricular depolarization, and the T wave represents ventricular repolarization, just before the ventricles start to relax.

The mineral required with iron for the synthesis of hemoglobin is: A sulfur B iodine C copper D chromium

C In addition to its role in synthesizing hemoglobin, copper is a component of coenzymes in the electron transport chain and an enzyme necessary for melanin pigment formation.

Iodine is necessary for the: A formation of hydrochloride in the stomach. B synthesis of hemoglobin. C synthesis of thyroid hormones. D potential conduction of nerve and muscle action.

C Iodine is required by the thyroid gland to synthesize thyroid homones, which regulate metabolic rate.

Lodine is used to treat: A high cholesterol B migraines C arthritis D herpes zoster

C Lodine [etodolac] is a nonsteroidal anti-inflammatory analgesic often prescribed to treat the pain associated with arthritis and other joint-related inflammatory processes.

Fosamax, a bisphosphonate used to treat osteoporosis, is often prescribed at a dose of: A 70 mg per day. B 70 mg per month. C 70 mg per week. D 7 mg per week.

C Originally prescribed at 10 mg per day for the treatment of osteoporosis, Fosamax was found to be effective at one combined dose of 70 mg per week.

Rifampin is used to treat: A gout B syphilis C tuberculosis D gonorrhea

C Rifampin is prescribed to eliminate Mycobacterium tuberculosis, the bacteria that causes tuberculosis. It is also given in conjunction with other drugs to combat Neisseria meningitidis.

Adrenal insufficiency is confirmed by which group of laboratory tests? A BUN, creatinine, and serum aldosterone B fasting glucose, CSF analysis, and electrolytes C serum cortisol, serum ACTH and plasma renin activity D CBC, electrolytes, and brain natriuretic peptide

C Routine lab studies for suspicion of adrenal insufficiency include electrolytes, fasting blood sugar, serum ACTH, plasma renin activity, serum cortisol, and serum aldosterone. The diagnosis of adrenal insufficiency is confirmed by a serum cortisol concentration less than 18 mcg/dL in the presence of an elevated serum ACTH concentration and plasma renin activity, or a concentration lower than that level obtained 60 minutes following cosyntropin administration.

An infectious disease specialist would order serological studies to: A determine antibiotic sensitivity of the infectious agent. B culture bacteria in the patient's blood. C determine the presence of antigens or antibodies in the patient's blood. D monitor peak and trough antibiotic levels.

C Serology is the study of antigens and antibodies in the serum. Antigens (proteins associated with an infectious agent) or antibodies (proteins made by the body directed against specific infectious agents) appear in the blood in the course of an infection and give the physician information about the infecting agent and how the body is responding to that infection. Some antibiotics, such as gentamicin, must be monitored carefully to maintain safe and therapeutic concentrations in the blood, and these antibiotic levels are referred to as peak (the highest concentration) and trough (the lowest concentration).

In ultrasonography, which term would be used to describe material that produces sound waves of higher amplitude than the surrounding structures? A hyperresonant B hyporesonant C hyperechoic D hypoechoic

C Sonographic echoes from a given anatomical structure are said to be "hyperechoic" when the sound waves return or bounce back at higher amplitude, or density, than the surrounding structures.

The Kurtzke disability score is used to evaluate patients with: A muscular dystrophy B cerebral hemorrhage C multiple sclerosis D Legg-Perthes disease

C The Kurtzke disability score is a two-part scoring system used to evaluate patients with multiple sclerosis. Part one evaluates functional systems (pyramidal, cerebellar, brain stem, sensory, bowel and bladder, visual, mental, and other). Part two is a disability status scale from 0 to 10.

Which test requires the use of a tuning fork? A Romberg B Dix-Hallpike C Rinne D Valsalva

C The Rinne and Weber tests utilize a tuning fork to distinguish between conductive and neurosensory sources of hearing loss. The Romberg test is used to evaluate equilibrium, Dix-Hallpike test to evaluate the source of vertigo, and the Valsalva maneuver to test the mobility of the tympanic membrane.

The ciliary body is found in the: A small intestine B ear C eye D nerves

C The ciliary body is the thickest portion of the vascular tunic, one of three tunic layers of the wall of the eyeball. It extends from the ora serata to a point just behind the sclerocorneal junction. The ciliar body consists of the ciliray processes and the ciliary muscle.

The patient has hirsutism and oligoovulation, consistent with polycystic ovarian syndrome. What can be said about this patient? A The patient would be a good oocyte donor. B She is probably being treated for cancer. C She is probably being treated for infertility. D The patient ovulates late in her cycle.

C The prefix oligo- means few or infrequent, so oligoovulation means infrequent ovulation. Polycystic ovarian syndrome is a common cause of infertility.

Which of the following contains a prepositional phrase? A He asked for either water or milk. B Dr. Jones, an expert pediatrician, was able to treat the child. C The lesion on his right upper thigh was biopsied. D My goodness!

C The prepositional phrase is on his right upper thigh. Prepositional phrases start with a preposition (e.g. on, to, toward, after, before) and usually end with a noun or pronoun. Prepositional phrases modify (help describe) the noun or the verb.

The scalene muscles are located in the: A forehead B hand C neck D abdomen

C The scalene muscles originate on the transverse processes of the third through sixth cervical vertebrae and insert at the first and second ribs. These muscles facilitate flexion/rotation of the neck and assist in inspiration.

Which is the secondary lymphoid organ that protects the body from blood-borne pathogens? A liver B bone marrow C spleen D adrenal glands

C The spleen functions in immunity as a site of B cell proliferation into plasma cells. Cytotoxic T cells leave lymphoid tissue to meet a foreign antigen, but B cells stay put. In the presence of a foreign antigen, specific B cells in lymph nodes, the spleen or lymphoid tissue in the GI tract become activated and differentiate into plasma cells that secrete specific antibodies, which then circulate in the lymph and blood to reach the site of invasion.

Which of the following is a malignant tumor of the eye that is known to be hereditary? A glioblastoma B neuroblastoma C retinoblastoma D pheochromocytoma

C This tumor arises from immature retinal cells and accounts for 2% of childhood malignancies.

Trichomonas vaginalis is a/an: A fungi B bacteria C protozoa D virus

C Trichomonas vaginalis is a flagellate protozoon that is pear-shaped and is pathogenic to the genitourinary tract. In woman, it lives only in the vagina and the Skene glands of the urethra.

Each of the following mature cells has a nucleus except: A lymphocyte B monocyte C erythrocyte D basophil

C Unlike white blood cells, red blood cells lack a nucleus and other organelles and can neither reproduce nor carry on extensive metabolic activities.

An x-ray study using contrast material to visualize the urinary bladder, ureters and urethra while actually passing urine is called a (an): A ultrasound study. B KUB C VCUG D renal angiograph

C VCUG (voiding cystourethrogram) is a study of the urinary system using x-rays and contrast material to detect blockages or constriction of urine flow. A KUB also uses x-rays to visualize the kidney, ureters and bladder, but not while the patient is voiding. Renal angiography studies the vascular system in and around the kidneys to detect problems with blood flow. Ultrasonography uses sound waves as opposed to x-rays.

Indurated means: A red. B long-standing. C softened. D hardened.

D Indurated comes from the root word durus which means hard. The term indurated usually refers to soft tissues which have become extremely firm, yet not as hard as bone. Other words using this same root word include dura mater (tough membrane covering the brain) and epidural (outside the dural mater).

What is the small locus of modified cardiac muscle fibers located near the ostium of the coronary sinus that passes impuses toward the ventricle? A. Purkinje fibers B. sinoatrial node C. bundle of His D. atrioventricular node

D

What laboratory result suggests the presence of acute viral hepatitis? A. decreased blood glucose B. increased urine sodium C. elevated urine chloride D. elevated AST

D

What muscle is the arrow above pointing to? (picture of muscles of the back. pointing to v-shaped muscles below the neck and between the scapulae. A. deltoid B. teres major C. sternocleidomastoid D. trapezius

D

What would be a treatment course for acute pancreatitis? A. emergency laparotomy B. tube feeding with fluid restriction, modified bed rest C. bed rest and modified diet D. fluid resuscitation, pain management

D

Which abbreviation refers to software used by physicians for direct entry of patient treatment information? A. CDR B. PACS C. OCR D. CPOE

D

Which are examples of correct classification of cervical intraepithelial neoplasias? A. CIN1, CIN2 B. CIN-A, CIN-B C. CIN-I, CIN-II D. CIN-1, CIN-2

D

Which condition suggested by an elevated erythrocyte sedimentation rate? A. pernicious anemia B. diabetes mellitus C. influenza D. temporal arteritis

D

Which is a factor impacting SRT adoption and integration? A. patient consent B. government approval C. Joint Commission standards D. efficiency and skill set of the editor

D

Which of the following factors most influences a successful SRT implementation? A. the type of technology being implemented B. lack of efficient editing skills C. inability to resolve style conflicts D. inconsistency of a dictator's speech pattern

D

The suffix -algia means: A inflammation. B burning. C soothing. D pain.

D -algia, which means pain, is a common suffix used in medical terminology. Examples include cephalgia (head pain), causalgia (burning pain), arthralgia (joint pain), and neuralgia (pain along the distribution of a nerve).

A _________ scan is used to determine the ejection fraction of the ventricles of the heart. A CT B PET C HIDA D MUGA

D A MUGA [multiple-gated acquisition] scan involves computer analysis of radionuclide emissions from the heart. Comparison of ventricular volumes at the end of diastole and systole enables calculation of ejection fraction.

A benign encapsulated tumor of the nerve sheath is a: A neuroblastoma B neurocytoma C neurosarcoma D neurilemoma

D A neurilemoma is a benign encapsulated tumor of the nerve sheath. Neurilemomas, like neurofibromas, are a type of schwannoma. A neurosarcoma is a malignant tumour originating from the sheath of peripheral nerves. Neuroblastoma is the most common extracranial childhood cancer and is malignant. A neurocytoma is a rare, benign tumor, typically occurring in a lateral ventricle in the region of the foramen of Monro.

Which of the following is correct? A The father, but neither of his sons, have been diagnosed with sickle cell disease. B The father and his son has been diagnosed with sickle cell disease. C Neither the father nor his son have been diagnosed with sickle cell disease. D The father and his son have been diagnosed with sickle cell disease.

D A subject with two nouns joined by the word and takes a plural verb (have). Subjects joined by neithernor are considered singular and therefore take the singular verb (has). Option D is not correct because the subject (the father) is singular. The dependent clause but neither of his sons is not part of the subject and should not be considered when determining verb agreement.

Which type of leukemia is associated with large numbers of mature lymphocytes in the circulating blood stream and is typically seen in the elderly? A acute myelogenous leukemia (AML) B acute lymphocytic leukemia (ALL) C chronic myelogenous leukemia (CML) D chronic lymphocytic leukemia (CLL)

D AML and ALL are associated with high numbers of circulating immature cells (either myeloblasts or lymphoblasts). CML and CLL are slowly progressive illnesses, the latter found typically in the elderly. All forms of leukemia are treated with chemotherapy to prevent cell division.

Which of the following ligaments are not found in the pelvis? A broad B round C cardinal D cruciate

D All of the ligaments except the cruciate ligaments are found in the pelvic area. The cruciate liagaments, damage to which has ended many a football player's career, are found in the knee.

Which cranial nerve is also known as the hypoglossal cranial nerve? A II B IV C X D XII

D All of the numbered cranial nerves are also associated with a nominal notation: I - olfactory, II - optic, III - oculomotor, IV - trochlear, V - trigeminal, VI - abducens, VII - facial, VIII - vestibulocochlear, IX - glossopharyngeal, X - vagus, XI - accessory, and XII - hypoglossal.

The heart's typical response to ischemia is: A sudden cardiac arrest B accumulation of fluid around the heart C distended neck veins D chest pain

D Although long-term ischemia can lead to infarction and cardiac arrest, the typical response to ischemia is chest pain, or angina pectoris.

Which of the following sentences contains the correct transcription of a compound modifier? A The work was back breaking. B She was a lovely looking female. C The child was well-developed and well-nourished. D The new nurse was quick thinking in a crisis.

D An adverb coupled with a participle only takes a hyphen when it precedes the noun it is modifying. In this sentence, the word quick functions merely as an adverb that interrupts and modifyies the verb phrase was thinking. Noun/participle modifiers (back-breaking) require a hyphen whether preceding or following their nouns. Compound modifiers containing an adjective ending in -ly (lovely-looking) always require a hyphen. Again, adverb/participle modifiers (well-developed, well-nourished) only take a hyphen when they precede the noun they modify, not when they follow it.

_____________ refers to generalized edema of the entire body. A ascites B dependent edema C angioneurotic edema D anasarca

D Anasarca represents an effusion of serum into the subcutaneous cellular substance, often resulting in massive generalized edema.

The azygos vein terminates at what vascular structure? A pulmonary vein B pulmonary artery C aorta D superior vena cava

D Arising from the right ascending lumbar vein or the inferior vena cava, the azygos vein ascends through the aortic orifice of the diaphragm, lies in the posterior mediastinum, and terminates in the superior vena cava.

BRCA-1 is a/an: A blood test for metastatic disease B estrogen receptor C tumor grading system D gene linked to breast cancer risk

D BRCA-1 is a breast cancer susceptibility gene that was first identified in 1994. People carrying a mutation (abnormality) in this gene are at an increased risk of breast or ovarian cancer. The normal gene plays a role in repairing breaks in DNA. However, when the gene is mutated it is thought that this repair function may become disabled, thus leading to more DNA replication errors and cancerous growth.

Brunner's glands are also known as the: A intestinal glands B pancreatic glands C microvilli D duodenal glands

D Brunner's glands secrete alkaline fluid to buffer stomach acids and mucus for protection and lubrication.

The drug Clomid is prescribed for: A migraines B weight loss C glaucoma D infertility

D Clomiphine citrate, or Clomid, to induce ovulation, correct irregular ovulation, to help increase egg production, and to correct luteal phase deficiency. It tricks the pituitary gland into producing more FSH and LH, thus stimulating the ovary to produce more eggs and follicles.

"The patient is 2 weeks status post left hemispheric stroke. She is eating well and maintaining weight, although she still has right-sided paralysis and __________." Which of the following would most appropriately complete this sentence? A dyscrasia B dysphagia C ectasia D dysphasia

D Dysphasia and dysphagia are the two most reasonable answers, but dysphasia (difficulty speaking) is the more likely answer. Dysphasia is caused by neurological damage, especially strokes on the left side of the brain. Dysphagia (difficulty swallowing) is less likely, since the physician indicated she is eating well.

Excessive development of hair or abnormal distribution of hair is known as: A trichomegaly B alopecia C hyperfollicularism D hirsutism

D Hirsutism is the condition referred to here and is often familial. In women, it can be associated with increased androgens in the blood or other more serious conditions, such as congenital adrenal hyperplasia or tumors of ovarian or adrenal tissue.

The root word ictal means: A yellow B inside C eye D seizure

D Ictal (ictus) pertains to a stroke or seizure and should not be confused with icterus, meaning jaundice. Preictal and postictal are common terms using this root word and refer to the time immediately preceding and following a seizure. Lamictal, a prophylactic medication for epilepsy, also uses this root word.

Which of the following growth factors is responsible for stimulating the development of neutrophils? A Interleukin-5 B Interleukin-7 C M-CSF D G-CSF

D Interleukin-5 is responsible for stimulating the development of eosinophils, Interleukin-7 for the development of lymphocytes, and M-CSF (Macrophage Cell Growth Factor) for the development of monocytes and macrophages. It is G-CSF (Granulocyte Cell Growth Factor) that is responsible for neutrophil development.

Which condition is associated with loop diuretics and typically requires the patient to take a supplement? A hyponatremia B hypernatremia C hyperkalemia D hypokalemia

D Loop diuretics work at the proximal and distal tubules and at the loop of Henle. They work to block the reabsorption of sodium into the bloodstream; by doing so, they block the reabsorption of much-needed potassium, thus requiring the use of a potassium supplement to combat hypokalemia.

Which of the following is transcribed correctly? A ½ mL B Synthroid .2 mg C 156.5 pounds D 0.5 mL

D Metric measurements are always reported in decimals whereas non-metric units of measure, such as pounds and inches, are written with common fractions. Decimals less than one should be written with a leading zero, especially for medications (e.g., Synthroid 0.2 mg) so that the decimal is not overlooked and the dosage mistaken for 2 mg (resulting in a 10-fold error).

A patient diagnosed with MAC is likely to have what underlying disorder? A lupus erythematosus B hepatitis B C tuberculosis D HIV/AIDS

D Mycobacterium avium complex (MAC) is among the most common bacterial infections of people with HIV/AIDS and is found in water, dust, soil and bird droppings. Normally found in small quantities in a healthy person, the bacteria proliferate in the immuno-compromised. It is most likely to be found in patients with a CD4 cell count below 50 and at least one other opportunistic infection.

The genus Mycobacterium includes: A parasites which cause pneumonia. B viruses which cause pneumonia. C fungi which cause upper respiratory infections. D the causative agent of tuberculosis.

D Mycobacterium tuberculosis is the bacterial causative agent of tuberculosis (TB). The nomenclature for mycobacteria as well as mycoplasma may be misleading, because the prefix myco means fungus, yet neither mycobacteria nor mycoplasma are fungi. Mycobacteria may also be referred to as acid-fast bacteria (as opposed to gram-positive or gram-negative bacteria). Acid-fast bacteria differ from other bacteria in that they will retain a red dye even when rinsed with acid alcohol. Mycoplasma species share characteristics with fungi and bacteria but are classified as neither. They are considered to be parasites, and Mycoplasma pneumoniae is the most common cause of community-acquired pneumonia, often referred to as "walking pneumonia."

Which of the following tests is used to assess the integrity of the peripheral nerves? A electronystagmogram B EEG C MCV D NCV

D NCV is the abbreviation for nerve conduction velocity. Damaged nerves often show decreased conduction and therefore decreased velocity of the electrical impulse traveling the course of the nerve. An NCV is often performed along with an EMG, or electromyelogram. MCV stands for mean corpuscular volume, which describes the size of a red blood cell. An ENG, or electronystagmogram, is used to assess nystagmus. An EEG, or electroencephalogram, is used to measure electrical potential of the brain using electrodes attached to the scalp.

Glomerulonephritis is also known as: A Brinton disease B Bristowe syndrome C Brown syndrome D Bright disease

D Named after Richard Bright, an English internist [1789-1858], the term is used to describe nonsuppurative nephritis characterized by albuminuria and edema, also known as glomerulonephritis.

The suffix -ipine is common to which of the following groups of generic drugs? A ACE inhibitors B beta blockers C corticosteroids D calcium channel blockers

D Nifedipine and amlodipine are examples of two common calcium channel blockers with this identifiable suffix.

Which of the following is used to treat hypertension? A Nolvadex B Noritate C naproxen D Norvasc

D Norvasc is a calcium channel blocker used to lower blood pressure by relaxing the walls of blood vessels and widening the internal caliber of vessels. Noritate is a topical antibiotic that is typically used to treat rosacea. Naproxen is a nonsteroidal antiinflammatory, and Nolvadex is an antineoplastic agent used to treat breast cancer.

Which of the following can be found in cartilage but not in bone tissue? A lacunae B protein fibers C blood vessels D chondroitin

D Of these, only chondroitin is found uniquely in the cartilage. The others are found in bone and other tissues of the body. It is chondroitin that is found in osteochondroitin, commonly taken now for bone and cartilage regeneration.

Which of the following would correctly denote a prescription for Diovan 80 mg with hydrochlorothiazide 12.5 mg to be taken twice a day? A Diovan HCT 80 mg/12.5 mg two per day B Diovan HCT 80/12.5 2 per day C Diovan HCl 80 mg/12.5 mg 2 tablets per day D Diovan HCT 80 mg/12.5 mg 2 per day

D Option A is incorrect because numerals that stand out from the surrounding text and serve a functional purpose should not be spelled out. Option B is incorrect because it leaves 2 separate numbers running together (12.5 2 per day) which could be misread as 12.52. Option C is incorrect because HCl is the abbreviation for hydrochloric acid, not hydrochlorothiazide.

Which of the following is an oral contraceptive? A Ortho Evra B Premarin C Depo-Provera D Ortho Tri-Cyclen

D Ortho Tri-Cyclen is the correct answer. Depo-Provera and Ortho Evra are also contraceptives, but they are not administered orally. Premarin is used for hormone replacement postmenopause or to treat dysfunctional uterine bleeding.

Which of the following is an example of an active (ATP-requiring) mode of transporting substances across the plasma membrane of a cell? A osmosis B diffusion C filtration D phagocytosis

D Osmosis, diffusion, and filtration are all passive processes, whereby substances are moved across the plasma membrane via their own kinetic energy, their compatibility with the permeable characteristics of the plasma wall, or the natural movement from areas of high concentration to low concentration. Only phagocytosis, which requires the pseudopods of the cell to expend energy engulfing the particles and bringing them into the cell, is considered an active transport process.

"Her affect is anxious. She admits that she does perseverate about her family's problems and is having difficulty sleeping." In this context, perseverate means: A she separates herself from her family's problems. B she is oblivious to her family's problems. C she blames herself for her family's problems. D she has persistent thoughts about her family's problems.

D Perseverate comes from the Latin term persevero meaning to persevere, persist. Perseveration can be a symptom of anxiety and/or depression. The patient experiences the same thought over and over, which interferes with activities of daily living and/or sleep.

Which of the following would be a normal finding in a person recovering from anemia due to a traumatic bleed? A blasts B sickle cells C schistocytes D reticulocytes

D Reticulocytes are immature red cells. They often appear in the peripheral blood when the marrow is trying to quickly raise the red cell count in the peripheral blood. A high reticulocyte count would be worrisome under normal circumstances, but is a positive sign of recovery for individuals who have recently suffered a low hematocrit. Schistocytes are pieces of red cell membranes; they are often the result of mechanical shredding of red cells caused by a prosthetic heart value or disseminated intravascular coagulation. Sickle cells are the result of an inherited chromosomal defect that alters the shape of hemoglobin. Blasts are extremely immature, undifferentiated white cells.

A patient who is para 3-1-0-4 has been pregnant at least: A once B four times C three times D twice

D Since it is possible that the patient had a single premature delivery and another delivery of triplets, the minimum number of times she could have been pregnant is two. The maximum number she could have been pregnant, if all 3 term births were from separate pregnancies, is 4.

Sjogren syndrome is a condition of the: A neurologic system B musculoskeletal system C vascular system D autoimmune system

D Sjogren syndrome is an autoimmune disease characterized by the abnormal production of extra antibodies in the blood that are directed against various tissues of the body. This particular autoimmune illness is caused by inflammation in the glands of the body. Sjogren syndrome that involves the gland inflammation but not associated with a connective tissue disease is referred to as primary Sjogres syndrome. Secondary Sjogren syndrome involves not only gland inflammation but is also associated with a connective tissue disease, such as rheumatoid arthritis, SLE or scleroderma.

In the condition tardive dyskinesia, the word tardive means: A occurring suddenly or without warning B uncontrolled C chronic D developing slowly or late-occurring

D Tardive dyskinesia develops over time in patients who have been on long-term antipsychotic medication, and it is characterized by involuntary, jerky movements of the muscles of the face, trunk, jaw, tongue and limbs.

Technetium is a/an: A gamma globulin B mineral C contrast material D radioisotope

D Technetium is a radioactive metallic element found in only trace quantities in its natural state. It is synthetically produced, and the isotope of technetium used in medicine is technetium 99m (Tc 99m), which has a half-life of about 6 hours.

Which of the following is transcribed correctly? A Her cervical motion is 50 per cent of normal flexion. She has a positive Sterling test on the right side. B Her cervical motion is 50% of normal flexion. She has a positive spur ring test on the right side. C Her cervical motion is 50% of normal flection. She has a positive Spurling's test on the right side. D Her cervical motion is 50% of normal flexion. She has a positive Spurling test on the right side.

D The Spurling test is used to assess cervical nerve root impingement. Per the Book of Style, eponyms are not written with an 's (eliminating answer A). Use of the percent symbol (%) is acceptable and typically easier to read. If necessary to spell out, use the combined form percent.

When the body's immune system detects an antigen, the response is to: A decrease the blood glucose B increase the blood glucose C release epinephrine D produce antibodies

D The body perceives any foreign substance to be an antigen, and it is the function of the immune system to attempt the production of antibodies in the presence of that antigen such that the body will be prepared and "stock-piled" in the event of future invasions by this same substance.

Which of the following would best complete this sentence: I would like him to see Dr. Smith for a _______ consultation regarding his postsurgical frequency and nocturia. A gynecologic B neurologic C nephrologic D urologic

D The clue in this case is "nocturia" which most often is a condition of the bladder. Although a gynecologist might address frequency and nocturia in a female, this patient is male. A nephrologist is more focused on the function of the kidneys.

Which of the following SSRI medications used to treat depression has a name derived from the Latin word for peace? A Celexa B Lexapro C Zoloft D Paxil

D The drug Paxil derives its name from the Latin pax, meaning peace. Pax Romana refers to the period of peace and tranquility that the Roman empire enjoyed from about 27 B.C. (with Augustus) until A.D. 180 -- the death of Marcus Aurelius.

Which of the following is not a stage of cellular mitosis? A prophase B anaphase C telophase D mitophase

D The four stages of cellular mitosis (or cellular division) are prophase, metaphase, anaphase and telophase whereby a single parent cell produces two chromosomally identical daughter cells.

The urine of someone with rickets would reflect which of the following levels? A decreased calcium and decreased phosphate B elevated calcium and elevated phosphate C elevated calcium and decreased phosphate D decreased calcium and elevated phosphate

D The normal range of urine calcium is 100-250 mg/dL and the normal range of daily phosphate excretion is 0.9-1.3 g/dL. Calcium levels are decreased and phosphate excretion increased in rickets. Lowered calcium levels stimulates parathyroid hormone excess, resulting in renal phosphate loss.

Which of the following is a nonsterile procedure? A amniocentesis B cystoscopy C peritoneal dialysis D proctosigmoidoscopy

D The other three procedures require a sterile field that reduces and maintains areas as free from microorganisms as possible.

The normal range for arterial blood pH is: A 2.0 to 3.5 B .0% to 3.5% C 7.35 % to 7.45% D 7.35 to 7.45

D The pH of blood is maintained in a very tight range of 7.35 to 7.45. Even slight changes in blood pH have ominous consequences. Answer C is incorrect because pH is not reported as a percent. The pH is a measure of hydrogen ion concentration, and pH stands for "potency of hydrogen." A pH of 7 is neutral; below 7 is acidic, and above 7 is alkaline.

The procedure that sutures the fundus of the stomach around the gastroesophageal junction to treat gastroesophageal reflux is called: A cecoplication B gastroplication C esophagoplication D fundoplication

D The root word plico means fold. Terms using this root word describe procedures that involve folds or tucks. Gastroplication is a procedure to reduce the size of the stomach by suturing longitudinal folds. Esophagoplication repairs outpouches or dilations of the esophagus and cecoplication repairs outpouches or dilations of the cecum.

An anterior drawer test is performed on the: A jaw B shoulder C wrist D knee

D The test is performed with the patient lying on the back with the knee in 90 degrees of flexion and the foot resting on the table. The examiner grasps the shin and places pressure on the tibial condyle, gently pulling the shin forward. Looseness and movement forward is indicative of a significant anterior cruciate ligament (ACL) injury.

Legislative measures intended to provide legal protection for citizens and some healthcare personnel who administer voluntary emergency care are known as: A First Responder laws B Patient Privacy laws C Helping Hands legislation D Good Samaritan laws

D This is a fundamental medico-legal principle. The Good Samaritan laws protect not only patients from zealous bystanders but also protect certain medical personnel in situations where they provide off-duty medical intervention in an emergency setting.

Laboratory values include a total cholesterol of 157, HDL 77, triglycerides 287, AST 25, ALT 34, and CPK 134. Given these values, the assessment could include: A hypocholesterolemia B liver disease C hypercholesterolemia D hypertriglyceridemia

D Triglyceride values above 250 are considered elevated. Normal reference range for cholesterol is 150-200, ruling out hyper- or hypocholesterolemia. AST and ALT are within the normal range (less than 35) therefore do not indicate the presence of liver disease.

The urinary drug Ditropan would be prescribed to treat: A pain and burning on urination B pyuria C hematuria D urgency and frequency

D While the other conditions may also be present in a patient with frequency and urgency, Ditropan [oxybutynin chloride] is an anticholinergic/antispasmodic prescribed to relieve the symptoms associated with urinary urgency and frequency, particularly in patients with bladder-control problems.


Conjuntos de estudio relacionados

Business Law Chapter 8: Intellectual Property

View Set

NURS 142 Exam 2 Modules 7&10 NCLEX

View Set

Napoleon Bonaparte - Webquest Packet

View Set

The Life of Merlin Chapters 1-14

View Set

Ch 15: Assignment - Mutual and Exchange Traded Funds

View Set